*NURSING > EXAM > NUR 206 , TESTBANK Chapter 15- 28 : Information Technology in the Clinical Setting Cherry & Jacob: C (All)

NUR 206 , TESTBANK Chapter 15- 28 : Information Technology in the Clinical Setting Cherry & Jacob: Contemporary Nursing: Issues, Trends, and Management, 7th Edition

Document Content and Description Below

Chapter 15: Information Technology in the Clinical Setting Cherry & Jacob: Contemporary Nursing: Issues, Trends, and Management, 7th Edition MULTIPLE CHOICE 1. Consumers are concerned with s... ecurity issues related to their confidential health information being placed in an electronic health record (EHR). However, when the security of the EHR is compared with that of paper-and-pencil records, the EHR is: a. more secure. b. less secure. c. equivalent. d. not comparable with the paper-and-pencil record. ANS: A Computer-based patient record systems, such as EHRs, provide better protection than paper-based systems. The EHR allows only authorized users to view data, and access to records can be audited for inappropriate use. DIF: Comprehension REF: p. 271 2. During a search for the term informatics, when the nurse finds the domain “.edu,” the site is affiliated with a(n): a. government agency. b. commercial site. c. educational institution d. Internet service provider. ANS: C The domain of an educational institution is .edu. DIF: Knowledge REF: p. 276 3. When paper-and-pencil medical records are compared with computer-based records: a. paper-and-pencil records provide controls to determine who has viewed the health information. b. information contained in a paper-and-pencil record has the capability of being more in-depth than that found in computer-based records. c. patients have the right to know that the confidentiality of their records is strictly maintained, regardless of the type of medical record used. d. patients must sign for each item of information released on the computer record. ANS: C Regardless of the type of record used, the Health Insurance Portability and Accountability Act (HIPAA) protects the confidentiality of the patient’s medical information and imposes legal consequences for those who breech confidentiality. DIF: Comprehension REF: p. 271 4. A nurse is preparing a scholarly publication on the prevalence of hepatitis A worldwide. The most efficient and effective means of conducting an Internet search to gather information for this publication is to use: a. a search engine such as Google or Yahoo. b. a consumer health website. c. a decision support system. d. MEDLINE database. ANS: D MEDLINE is one of the scientific and research scholarly databases, and it would be the most appropriate for use in gathering information for a scholarly publication. DIF: Comprehension REF: p. 276 5. A consumer is learning about electronic health records at a local health fair and states, “I am worried that someone can read my health information and I really don’t understand the difference between privacy and confidentiality.” The nurse explains that an example of confidentiality would be: a. a pledge that states, “I will hold matters pertaining to my patients in strict intimacy.” b. a patient who does not tell the physician that he has been treated for a sexually transmitted disease. c. a teenager who sustains a broken arm and in the emergency department and withholds information about her use of recreational drugs. d. locking medical records in cabinets to prevent unauthorized users from accessing patient information. ANS: A Confidentiality is keeping private the personal information that was given to a health care provider, unless others have a legitimate need to know. DIF: Application REF: p. 271 6. A physician has installed a computer-based patient records system. An outside care provider who requests medical information must obtain the patient’s signed consent and then is assigned a password to gain access to the medical information. A monthly audit is conducted to determine for whom and for what purpose patient records have been accessed. This protection is referred to as: a. privacy. b. confidentiality. c. security. d. data capture. ANS: C Security is the limitation of access to health care information through passwords and other precautions. DIF: Comprehension REF: p. 271 7. A nurse walks up to a computer in the hallway and presses the index finger to the sensor, thereby gaining access to patient data. A few moments later another nurse performs the same steps and is granted access. A visitor who is watching from a room walks over and places the index finger on the sensor, only to receive an “error and access denied” message. Security is being maintained by: a. robot technology. b. biometric technology. c. telehealth. d. ubiquitous computing. ANS: B Biometric fingerprint identification uses personal characteristics to allow access to health information. DIF: Comprehension REF: p. 277 8. A nurse who is teaching a class to introduce telehealth to the staff would include which example? a. A robot performs menial housekeeping chores for an invalid patient. b. A computer software program alerts the nurse or physician who is reviewing orders that an order for a new drug can cause synergy of the theophylline inhaler. c. A physician speaks into a computer, and the admission history is recorded and saved in the patient file. d. While a patient in Wyoming performs peritoneal dialysis, a nurse watches remotely from California to ensure that all steps are being followed correctly. ANS: D Telehealth is the delivery of care to a patient who is at a distance from the health care provider. DIF: Application REF: p. 273 9. An advanced practice nurse inputs into a computer software program the following clinical manifestations: open wound with tibia exposed, petechial hemorrhage, and temporary loss of consciousness. The computer diagnosis of fat emboli is generated by a system known as: a. decision support. b. telehealth. c. robotic technology. d. biometric technology. ANS: A Decision support systems are computer-based information systems that include knowledge-based systems designed to support clinical decision making. DIF: Comprehension REF: pp. 268-269 10. A nurse is preparing a presentation using different websites to collect information. The nurse is concerned that contact information and the author’s credentials are not listed for one of the websites reviewed. Which criterion required to establish a reputable website is missing? a. Authority b. Objectivity c. Usability d. Currency ANS: A Authority is the criterion that is related to the credentials and background that have prepared an author to publish on the subject. DIF: Comprehension REF: p. 276 11. A nurse is interested in locating reliable information concerning noninvasive blood glucose monitoring. Information is located, and the author is a scientist who conducted studies within the last year on the effectiveness of a particular noninvasive blood glucose monitor. The scientist received funding from a pharmaceutical company to support the studies. The URL indicates the pharmaceutical company site.com. The nurse is concerned about this information’s: a. authority. b. objectivity. c. accuracy. d. currency. ANS: B Sites sponsored by organizations such as pharmaceutical companies may influence the content. DIF: Comprehension REF: p. 276 12. A nurse providing care at the bedside receives an “alert” that a patient’s stat potassium level is 2.5 and digoxin (Lanoxin) is scheduled. The nurse holds the medication and prevents a possible complication. This feature of the Electronic Health Record is available through which core function of EHR? a. Order entry/order management b. Decision support c. Patient support d. Administrative support ANS: B Decision support provides reminders about preventive practices, such as immunizations, drug alerts for dosing and interactions, and clinical decision making. DIF: Comprehension REF: pp. 268-269 13. A nurse works on a unit where electronic health records (EHR) are being initiated and asks, “What is meant by ‘meaningful use’ standards that are in our education packet?” The best answer is that “meaningful use”: a. identifies a set of EHR proficiencies and benchmarks that EHR systems must meet to be certain that they are functioning to their maximum capacity and meeting this standard allows companies/organizations to qualify for funds to defray cost of the EHR from Medicare. b. refers to training competencies that all users must achieve to be able to access and transfer patient data/information. c. refers to a requirement that at least 50% plus one of all patients have data entered into the EHR. d. the requirement that rigorous confidentiality security is in place to protect all patient information from sources which have no right to the data. ANS: A Meaningful use is “A defined set of EHR capabilities and standards that EHR systems must meet to ensure their full capacity is realized and for the users (hospitals and physician practices) to qualify for financial incentives from Medicare.” DIF: Application REF: p. 271 14. A nurse is caring for a patient who is to receive an antibiotic drug that causes severe skin damage when infiltrated. The order reads, “infuse over 1 hour by portacath.” The nurse accesses the Personal Digital Assistant for software that lists the steps to access a portacath. The nurse is using: a. electronic health records. b. point-of-care technology. c. data management. d. telehealth. ANS: B Using a Personal Digital Assist device to access information at the bedside is considered point-of-care technology. The nurse was able to retrieve the steps for accessing a portacath electronically while remaining at the bedside. DIF: Comprehension REF: p. 273 15. The Institute of Medicine (IOM) (2003) recommends that EHR systems offer eight functionalities. A patient has a severe allergy to eggs and penicillin. Which of the eight functions of the EHR would address sharing this information? a. Health information and data capture b. Results/data management c. Provider order entry management d. Clinical decision support ANS: A The health information and data capture function includes information such as medical history, laboratory tests, allergies, current medications, and consent forms. DIF: Comprehension REF: p. 269, Table 15-1 MULTIPLE RESPONSE 1. A new nurse asks, “Since Electronic Medical Records can improve quality care by having seamless data available for a patient, why doesn’t everyone just replace paper and pencil charts”? Barriers to a universal health information infrastructure include the fact that: (select all that apply) a. competition from individual companies to build EMR prevent a universal infrastructure. b. cost is prohibitive even with federal funding for larger health care systems. c. preventive health reminders for immunizations and yearly screenings such as mammograms are used in clinical decision making. d. insurance companies have halted sharing of some patient data due to fear of law suits. e. the full capacity of EHRs has not been realized with only Stage 1 of 3 nearing completion. ANS: A, E It has been recommended that only a federal-based EMR would provide an infrastructure that allows access to comprehensive patient information. The first stage, years 2011 and 2012, forms the foundation for electronic data capture and information sharing. DIF: Comprehension REF: p. 271 2. A nurse interested in quality improvement tools performed a search for cause and effect diagrams using www.ishikawa.com. A page opened that provided images and templates for performing fishbone diagrams. Which type of search did the nurse conduct? a. Quick and dirty b. Advanced c. Brute force d. Link searching ANS: C Brute force is a method of searching where you type in what you think might logically be a web address and see what happens. DIF: Comprehension REF: p. 275 COMPLETION 1. Software programs that process data to produce or recommend valid choices are known as . ANS: decision support systems Decision support systems use software programs that process data to produce or recommend decisions by linking with an electronic knowledge base. DIF: Knowledge REF: p. 269 Chapter 16: Emergency Preparedness and Response for Today’s World Cherry & Jacob: Contemporary Nursing: Issues, Trends, and Management, 7th Edition MULTIPLE CHOICE 1. Nurses and community officials are working together to ensure that churches and schools have needed supplies to provide shelter for large numbers of individuals in the event of a natural or manmade disaster. These activities represent which phase of a disaster continuum? a. Preparedness b. Relief response c. Recovery d. Crisis intervention ANS: A Every disaster response begins as a local event known as the preparedness phase, which consists of planning, preparedness, prevention, and warning. DIF: Comprehension REF: p. 285 2. A nurse at a school notices that several students have “blisters” on their bodies. Further investigation reveals that a terrorist incident has occurred, causing smallpox. If the chemical, biologic, radiologic, nuclear, and explosive (CBRNE) agent categories are used, this incident would be classified as: a. chemical. b. biologic. c. radiologic. d. nuclear. ANS: B The biologic category refers to diseases such as plague or smallpox. DIF: Comprehension REF: p. 284, Table 16-1 3. The Metropolitan Medical Response System (MMRS): a. is totally financed by the federal government national disaster fund. b. consists of responders who have obtained specialized training and equipment to deal with mass casualty events. c. has a storehouse of medications and antidotes to be used during response in times of national emergencies. d. arranges for patient admissions to federal hospitals. ANS: B The MMRS responds to disaster with trained individuals who have expertise in this type of situation and who have the equipment required to be effective. DIF: Comprehension REF: p. 286 4. A community in the New Madrid fault zone experiences an earthquake resulting in injuries from propelled objects and abrasions for many victims. The local supply of antibiotics is quickly exhausted. Local authorities would contact the: a. Commissioned Corps Readiness Force. b. Strategic National Stockpile. c. Department of Homeland Security. d. local Young Men’s Christian Association (YMCA). ANS: B The Strategic National Stockpile provides antibiotics, antidotes, and medical and surgical items when local and state supplies have been exhausted. DIF: Comprehension REF: p. 287 5. During the relief response phase of a disaster resulting from a “dirty bomb”: a. treatment for burns and poisoning is provided for victims. b. emergency plans are coordinated between agencies. c. reconstruction of destroyed facilities and homes begins. d. food stores are collected for potential victims. ANS: A During the relief response phase of a disaster, emergency responders provide assistance to victims and stabilize the scene; with a dirty bomb, radioactive material causes burns and poisoning. DIF: Analysis REF: p. 288 6. A nurse learns of a mass casualty disaster following a known terrorist attack. On arriving at the scene, the nurse knows that: a. the response of local hospitals will be dictated by the federal government. b. the same ground rules practiced in other settings and during smaller crises will be applicable. c. the least experienced nurses will be assigned to triage low-risk victims and victims who have no chance of survival. d. multiple incident commanders ensure a quick, effective response. ANS: B The fundamentals of nursing applied to other settings and situations can be used in a disaster. DIF: Comprehension REF: p. 283 7. Nurses caring for the victims of a mass casualty incident: a. determine the common terminology to be used by hospitals and participating agencies. b. take charge of communicating with the news media. c. determine whether there is a credible threat of a terrorist attack. d. give priority for care to those with the greatest chance of survival rather than those most critically ill. ANS: D Care is shifted from categorizing patients at low, intermediate, and critical risk to using resources to serve those with the greatest likelihood of survival. DIF: Comprehension REF: p. 283 8. During a community health fair the disaster medical assistance team (DMAT) informs participants that every community must be ready to provide disaster care. A participant asks, “In a disaster, the local community cannot possibly be effective, so why not have a plan to call federal agencies immediately to provide relief?” The correct response by the DMAT is: a. “Unless known terrorist activities involving mass destruction occur, the federal government does not become involved.” b. “The community is essentially the ‘first responder’ to any disaster.” c. “The preparedness phase of a disaster is the responsibility of the community, the relief response phase is assigned to state agencies, and the recovery phase is the responsibility of federal agencies.” d. “Unless local health care facilities are incapacitated, state and federal agencies will withhold assistance.” ANS: B Each disaster begins locally, and each community responds first and receives assistance from state and federal agencies when local resources are not adequate for the situation. DIF: Comprehension REF: p. 285 9. A nurse who is conducting a staff in-service on the phases of a disaster continuum teaches participants that, during the impact/response stage, activities focus on: a. community awareness in anticipation of a terrorist attack or natural disaster. b. determining the effectiveness of the disaster medical assistance team (DMAT). c. the use of an all-hazards approach. d. initiating response activities. ANS: D Response activities during the relief response phase consist of immediate actions to save lives and meet basic human needs. DIF: Comprehension REF: p. 288 10. Following a terrorist attack, victims are exhibiting posttraumatic stress syndrome, and care providers are exhibiting compassion fatigue. Which federal response system should be initiated? a. Strategic National Stockpile b. Metropolitan Medical Response System (MMRS) c. Commissioned Corps Readiness Force d. National Disaster Medical System ANS: B The MMRS is concerned with deploying trained responders who are able to provide mental health care for victims and health care providers. DIF: Comprehension REF: p. 286 11. The crisis communication officer may first inform the public or health care facility of a disaster or an act of terrorism. This representative has the responsibility to: a. contain the facts to within the administration group. b. incite the public to quickly take cover and obtain emergency supplies. c. provide understandable and straightforward facts about the event within the facility and possibly to the news media. d. inform the public that no information can be released until it has been confirmed by state and federal agencies. ANS: C The crisis communication officer is the first contact for patients, families, and employees within the facility or news media, so they may better understand the situation and know how to react and protect themselves. DIF: Comprehension REF: p. 289 12. The disaster medical assistance team works quickly to contain contaminants from a chemical plant explosion. Afterward, personnel undergo a special process to remove harmful chemicals from equipment and supplies. This removal process is known as: a. containment. b. decontamination. c. triage. d. scene assessment. ANS: B Decontamination is the physical process of removing harmful substances from personnel, equipment, and supplies. DIF: Knowledge REF: p. 280 13. A group of local volunteers respond to a tornado. Volunteers have completed an emergency response course and are able to assist with triage of injured citizens. They also participate in local health fairs to teach residents how to react during tornadoes. The responders are members of the: a. Medical Reserve Corps (MRC). b. Metropolitan Medical Response System (MMRS). c. National Disaster Medical System (NDMS). d. Commissioned Corps Readiness Force (CCRF). ANS: A The MRC are local volunteers trained to respond to local emergencies. DIF: Comprehension REF: p. 285 14. Troops from the United States participating in a peace mission in a foreign country were the victims of suicide bombers and many soldiers were evacuated back home to receive specialized medical care. The nation’s medical responses will be augmented by: a. the federally coordinated National Disaster Medical System. b. local homeland communities where troops receive care. c. the Medical Reserve Corps, which organizes and utilizes public health, nursing, medical, and other volunteers. d. the National Incident Management System, which guides government, nongovernmental organizations, and the private sector to work seamlessly during disaster situations. ANS: A The National Disaster Medical System supplements care for casualties evacuated back to the United States from overseas and federally declared disasters including national disasters, major transportation accidents, technologic disasters, and acts of terrorism. DIF: Comprehension REF: pp. 286-287 15. A nurse is informed that the Federal Bureau of Investigation has determined that a bomb has been detected and is in the possession of a known terrorist group. The government buildings in the local community are the target. This situation is termed a(n): a. all-hazards approach. b. biologic event. c. credible threat. d. natural disaster. ANS: C A credible threat is a situation in which the Federal Bureau of Investigation (FBI) determines that a terrorist threat is probable and verifies the involvement of a weapon of mass destruction in the developing terrorist incident. DIF: Comprehension REF: p. 280 16. The emergency response team responded to a terrorist attack where hundreds of people died following symptoms of chest tightness, palpations, seizures, and finally paralysis. A colorless odorless liquid known as Sarin (GB) was the agent, which is primarily inhaled with limited exposure through the skin. The concentration of Sarin has not been measured. What level is the minimum level of personal protection and safety equipment (PPE) that would be needed? a. A b. B c. C d. D ANS: B Level B requires a high level of respiratory protection, but less skin protection, providing a chemical splash–resistant suit with hood and self-contained breathing apparatus (SCBA). It provides maximum respiratory protection but less skin protection than level A equipment. DIF: Analysis REF: p. 289 17. A state is devastated by a tornado killing many people, destroying communication systems, utility services, homes, and medical facilities. The state requests immediate assistance from the U.S. Congress and from surrounding states. The affected state should first contact the: a. Emergency Management Assistance Compact (EMAC). b. Institute of Medicine (IOM). c. Red Cross. d. Strategic National Stockpile. ANS: A The EMAC is an organization authorized by the U.S. Congress through which a state impacted by a disaster can request and receive assistance from other member states quickly and efficiently. DIF: Comprehension REF: p. 280 18. A nurse is interested in learning the phases of the disaster continuum and realizes it has many similarities to the nursing process. To better understand the phases of a disaster, which is true when comparing the phases of the disaster continuum to the nursing process? a. The preparedness phase of the disaster continuum is consistent with the assessment and planning steps of the nursing process. b. The recovery phase of the disaster continuum is consistent with the planning step of the nursing process. c. The recovery phase of the disaster continuum is consistent with the implementation step of the nursing process. d. The response relief phase of the disaster continuum is consistent with the evaluation step of the nursing process. ANS: A The preparedness phase requires assessing possible needs of the community and planning appropriate interventions and is consistent with the assessment and planning steps of the nursing process. DIF: Comprehension REF: p. 285 19. In the preparedness phase for disasters, the community plans for a possible terrorist attack using anthrax as the weapon of destruction. What treatments and/or preparations would be needed? a. Vaccines and Level B Personal Protection Equipment (PPE) b. Treatment for burns, decontamination, and Level A PPE c. Social distance determination, decontamination for radioactive fallout d. Identify and detect incendiary devices, treatment for burns and propellants ANS: A Anthrax is a biologic weapon and requires Level B protection since it is a known agent and can be carried in wind or surfaces. Timing of treatment is critical and vaccines are available. DIF: Analysis REF: pp. 282-284, Table 16-1 MULTIPLE RESPONSE 1. Health care professionals have been activated to respond to a disaster, and the registered nurse who is coordinating the effort realizes that: (select all that apply) a. in the event of a mass casualty incident, care is prioritized to those who have the greatest chance of surviving. b. communities should use their own resources first to attempt to stabilize and organize the response. c. state assistance occurs any time a disaster occurs, regardless of the community’s resources. d. the emergency operating plan developed by one central agency rather than individual facilities should be put into operation. e. strict protocols regarding the use of resources must be followed. ANS: A, B Care is shifted to doing the most good for the most people. Efforts begin at the local level. DIF: Comprehension REF: pp. 283-284 2. When teaching community preparedness for a community group, the nurse explains that components of the National Disaster Medical System (NDMS) provide for: (select all that apply) a. a nationwide bomb disposal squad team for the rapid removal of explosive devices. b. teams of health care providers who are experts and have specialized supplies and equipment. c. structures for patient evacuation from the disaster area to an unaffected area. d. arrangements for hospitalization in federal and volunteer nonfederal acute care hospitals. e. providing mental health care for the community, for victims, and for health care providers. ANS: B, C, D The NDMS provides specially trained teams of people along with equipment designed for disaster relief. The NDMS is responsible for removing patients from unsafe to safe areas. The NDMS coordinates efforts to evacuate victims to federal or nonfederal volunteer hospitals that can care for disaster victims. DIF: Comprehension REF: p. 286 COMPLETION 1. The emergency preparedness term that is used to describe the process of limiting the emergency situation within a well-defined area is . ANS: containment Containment is correct because the focus is to prevent the agent that caused the disaster from spreading. DIF: Knowledge REF: p. 280 2. The term used during a pandemic disaster that refers to the attempt to contain germs by limiting socialization and personal interactions is . ANS: social distancing The term social distancing refers to the attempt to keep people as far apart as possible so as to limit the possibility of spreading germs. DIF: Knowledge REF: p. 291 Chapter 17: Nursing Leadership and Management Cherry & Jacob: Contemporary Nursing: Issues, Trends, and Management, 7th Edition MULTIPLE CHOICE 1. In an attempt to persuade employees to bargain for another type of health insurance, a handout is circulated that describes the present employees’ health care insurance as being insensitive, limiting choices of care providers, and providing inferior care. This reflects which aspect of Lewin’s planned change? a. Unfreeze b. Move c. Refreeze d. Acceptance ANS: A Unfreeze is correct because the change agent promotes problem identification and encourages awareness of the need for change. In alignment with Lewin’s stages of change (unfreezing, moving, and refreezing), education and involvement are keys to successful change. People must believe that improvement is possible before they will be willing to consider change. DIF: Comprehension REF: p. 311 2. One difference between a leader and a manager is that a: a. leader has legitimate authority. b. manager motivates and inspires others. c. manager focuses on coordinating resources. d. leader focuses on accomplishing goals of the organization. ANS: C The terms leadership and management are often used interchangeably, and it is difficult to discuss one without discussing the other. However, these roles have specific traits unique to themselves. The manager is the coordinator of resources (time, people, and supplies) needed to achieve outcomes. DIF: Comprehension REF: p. 296 3. The first step in the nursing process and in the problem-solving process is to: a. identify the problem. b. gather information. c. consider the consequences. d. implement interventions. ANS: B The nursing process, which is familiar to nurses who address patient care needs, can be applied to all management activities that require decision making and problem-solving. As in the nursing process and the problem-solving process, one must first gather information about the problem or situation. DIF: Comprehension REF: pp. 309-310 4. An explosion just occurred at the local factory, and hundreds of employees have sustained varying degrees of injury. Which type of nursing leadership is most effective in this situation? a. Autocratic b. Democratic c. Laissez-faire d. Referent ANS: A The dynamics of the situation demand that the leader take control and direct employees to specific actions in response to the emergency. DIF: Comprehension REF: p. 300, Box 17-3 5. An RN with excellent assessment and psychomotor skills would derive power on the basis of which source? a. Rewards b. Coercion c. Expert d. Legitimate ANS: C Seven primary sources of power are known. Expert power is based on knowledge, skills, and information. DIF: Comprehension REF: p. 297 6. Managers who exhibit an authoritative behavioral style are most likely to use which source of power? a. Informal b. Expert c. Coercive d. Reward ANS: C Seven primary sources of power are known. Coercive power is based on fear of punishment or failure to comply. Coercive power fits well into the authoritative behavioral style because authoritative managers dictate the work with much control, usually ignore the ideas or suggestions of subordinates, and provide little feedback or recognition for work accomplished. DIF: Application REF: p. 300, Box 17-3 7. Florence Nightingale is attributed with being intelligent (she developed statistical methods to evaluate health care), dependable (she often worked long hours to care for the injured), and ambitious (she fought against society’s perception of nursing). Those who depict her as a leader on the basis of these qualities are practicing which leadership theory? a. Trait b. Chaos c. Bureaucracy d. Organizational ANS: A Leadership trait theory describes intrinsic traits of leaders and is based on the assumption that leaders were born with particular leadership characteristics. Other traits found to be associated with this leadership theory include intelligence, alertness, dependability, energy, drive, enthusiasm, ambition, decisiveness, self-confidence, cooperativeness, and technical mastery. DIF: Comprehension REF: p. 298 8. A nurse manager is concerned with restocking the emergency cart, creating the staff schedule, requesting floor stock from pharmacy, and checking the orders on patient charts. Which type of leader accurately describes this nurse? a. Transactional b. Situational c. Transformational d. Contemporary ANS: A The transactional leader is concerned with the day-to-day operations of the facility. DIF: Comprehension REF: p. 298, Box 17-1 9. According to the unit’s policy for call-ins, a nurse is suspended for 3 days because of excessive call-ins that occur within 15 minutes of shift change. The nurse states, “You are unfair to me.” Which theory would disprove the nurse’s statement? a. Authoritative b. Closed systems c. Open systems d. Trait ANS: A Autocratic/authoritative management style revolves around the assumption that authority confers the right to issue commands within an organization on the basis of impersonal rules and rights, by virtue of the management position rather than any trait ascribed to the person who occupies that position. Other characteristics include the following: Impersonal rules govern the actions of superiors over subordinates, all personnel are chosen for their competence and are subject to strict rules that are applied impersonally and uniformly, and a system of procedures for dealing with work situations is in place. Represents the systems theory of the organization. DIF: Application REF: p. 300, Box 17-3 10. A nurse is reading about positive reinforcement with the goal of increasing staff motivation. Which action would demonstrate positive reinforcement? a. Every morning at shift change, thank each employee for an excellent job. b. Rotate a monthly “employee recognition award” among all employees on the unit. c. Wait until the annual performance review to recognize accomplishments. d. Give spur-of-the-moment recognition to an employee who has accomplished a goal. ANS: D To be effective, positive reinforcement should (1) be specific, with praise given for a particular task done well or a goal accomplished; (2) occur as close as possible to the time of the achievement; (3) be spontaneous and unpredictable (praise given routinely tends to lose value); and (4) be given for a genuine accomplishment. DIF: Application REF: p. 306 11. Which action represents the key management function of strategic planning? a. Determining that all nurses on the unit understand the current organizational philosophy b. Evaluating the communication process between the pharmacy and the nursing departments c. Monitoring data from the quality management initiative related to the last three orientation programs d. Developing a 5-year plan that will incorporate the clinical nurse leader as a part of all nursing units ANS: D A strategic plan is a written document that details organizational goals, allocates resources, assigns responsibilities, and determines time frames. The strategic plan generally looks 3 to 5 years into the future. DIF: Application REF: pp. 301-302 12. A hospital’s policy requires that all nurse managers must have a minimum of a bachelor’s degree in nursing. A BSN nurse new to the hospital has recently been hired as nurse manager for the oncology unit. An RN who has worked on this unit for many years is unable to be promoted to a nurse manager position because of his educational status and has been commenting to physicians and staff, “The new nurse manager has book sense but no leadership abilities.” What is the best approach that can be used by the new nurse manager who is attempting to gain the trust and respect of the nursing staff on the unit? a. Send memos to all staff except the upset nurse to invite them to a luncheon. b. Ask management to transfer the upset nurse to another unit. c. Assign the upset nurse to committees that do not directly affect that nursing unit. d. Acknowledge the clinical expertise of the upset nurse and clearly explain the expectations for teamwork and open, honest communication. ANS: D The best way for the new nurse manager to communicate with this employee, who may be an informal leader, is to show respect for the individual’s clinical expertise and experience through clear and direct communication. The new nurse manager should attempt to identify the staff nurse’s power as an informal leader, should involve him and other staff members in decision-making and change-implementation processes, and should clearly communicate goals and work expectations to all staff members. DIF: Application REF: pp. 297-298 13. A clinical nurse leader (CNL) enters the workforce and hopes to use her interdisciplinary skills to participate on a quality improvement committee. The coordinator of the quality group invites the CNL to join the group. Which type of power is demonstrated by the coordinator of the group? a. Coercive b. Transformational c. Laissez-fair d. Legitimate ANS: D The coordinator of the committee has an official position within the organizational committee. DIF: Comprehension REF: p. 297 14. A director of nursing (DON) asks the staff to list how their nursing unit can help the organization meet its goal to “provide quality patient care with attention to compassion and excellence.” An ad hoc committee is formed to develop a timeline of identified actions. The DON coaches the committee to reach desired outcomes. This DON is demonstrating which other role of leadership and management? a. Transactional b. Clinical consultant c. Corporate supporter d. Autocratic ANS: C The manager is embracing the mission of the organization by supporting achievement of goals noted in the mission statement. DIF: Comprehension REF: p. 311 15. A staff nurse provides care based on intuition and always seems to be in control of her personal and professional life—serving on the board of the state nurses association, serving as the nursing unit’s representative on the ethics committee, and coaching her daughter’s soft ball team. Many of the staff observes how she manages time and provides care. This nurse’s power comes from which type of power? a. Referent b. Legitimate c. Information d. Connection ANS: A Referent power comes from the followers’ identification with the leader. Referent leaders are admired and respected and able to influence other nurses because of their desire to emulate her. DIF: Comprehension REF: p. 297 16. A staff nurse states, “I really enjoyed having dinner with the Chief of Medical Staff and the President of the hospital. We hope to meet again soon.” Which source of power does this nurse possess? a. Expert b. Legitimate c. Connection d. Reward ANS: C Connection power results from knowing or associating with power people such as the upper administration. DIF: Comprehension REF: pp. 297-298 17. A nurse manager wants his nursing unit to be a place where all nurses want to work, where patient satisfaction is high, and care is innovative and interdisciplinary. Staff are encouraged to chair taskforces to improve quality of care and he counsels staff in areas of measuring patient outcomes. Other managers want to mimic this manager’s approach to improve their own units. This nurse is which type of leader? a. Transformational b. Transactional c. Laissez-faire d. Authoritative ANS: A Transformational leaders mentor followers through a vision and are admired and emulated. DIF: Comprehension REF: p. 298 18. A hospital recently learned that their scorecard did not meet the national benchmark for patient satisfaction and brought in a professional change agent to determine what their issues were and how they could improve their score. The agent collected data and recommended that nurses participate in interdisciplinary walking rounds and allow the patient and family to be participants. Nurses now round every shift and perform “huddles to update the team” as needed throughout the shift as part of best practices. Random visits are made to nursing units to ensure all nurses are participating and patients are interviewed for their involvement. This stage of Lewin’s change is: a. unfreeze. b. moving. c. refreeze. d. resistance. ANS: C In the refreezing stage, change becomes status quo and the agent reinforces until the change is part of the daily process as in the above situation. DIF: Comprehension REF: p. 311 19. A manager just finished the last annual performance review of the staff, reviews the unexpected expenditures for the month due to use of agency nurses, and shares the latest quality indicators with the staff nurses. This manager is performing which management function? a. Directing b. Planning c. Organizing d. Controlling ANS: D Controlling is the final management function. It includes performance evaluations, financial activities, and tracking outcomes of care to ensure quality. DIF: Comprehension REF: p. 307 MULTIPLE RESPONSE 1. Registered nurses who are entering the workforce will have expanded leadership responsibilities that include: (select all that apply) a. serving on interdisciplinary care teams. b. being competent to work in several areas independently when dictated by patient census. c. attending a meeting to plan advanced training for unlicensed assistive personnel. d. evaluating outcomes of care that are reported to a standing committee. e. managing units with higher acuity, shorter length of stay, and more diverse patients and staff. ANS: A, C, D, E The new nurses will be placed in many situations that require leadership and management skills: for example, managing a group of assigned patients, serving on a task force or committee, acting as team leaders or charge nurses, and supervising unlicensed assistive personnel and licensed vocational/practical nurses. Diverse patients have comorbidities and require complex interventions delivered during shorter stays with an ever-increasing diverse staff. The new nurses will be placed in many situations that require leadership and management skills, for example, managing a group of assigned patients, serving on a task force or committee, acting as team leaders or charge nurses, and supervising unlicensed assistive personnel and licensed vocational/practical nurses. The new nurses will be placed in many situations that require leadership and management skills, for example, managing a group of assigned patients, serving on a task force or committee, acting as team leaders or charge nurses, and supervising unlicensed assistive personnel and licensed vocational/practical nurses. The new nurses will be placed in many situations that require leadership and management skills, for example, managing a group of assigned patients, serving on a task force or committee, acting as team leaders or charge nurses, and supervising unlicensed assistive personnel and licensed vocational/practical nurses. DIF: Application REF: pp. 296-297 2. A nurse asks, “What is meant by ‘internal’ customers?” The correct response is: (select all that apply) a. insurance companies b. accreditors such as The Joint Commission c. X-ray technicians d. clinical pharmacist e. chief financial officer ANS: C, D, E Internal customers are employees of an organization at all levels. An x-ray technician is an example of an internal customer. Internal customers are employees of an organization at all levels. A clinical pharmacist is an example of an internal customer. Internal customers are employees of an organization at all levels. A chief financial officer is an example of an internal customer. DIF: Comprehension REF: p. 308 Chapter 18: Budgeting Basics for Nurses Cherry & Jacob: Contemporary Nursing: Issues, Trends, and Management, 7th Edition MULTIPLE CHOICE 1. A nurse manager plans the fiscal budget to include salaries for two RNs for two 12-hour shifts with a patient census of 6 in the short-stay observation room. The nurse manager reviews the budget report 3 months later and notes that the salary expenses are higher than was budgeted because of higher-than-planned RN staff salaries. This additional RN staff is necessary to meet patient care needs because the census has remained constant at 10 patients rather than the 6 projected when the budget was developed. The difference between the planned budget and the actual cost is known as: a. revenue. b. variance. c. monitoring. d. capital expenditures. ANS: B Variance is the difference between the planned budget and actual results; it can be a positive or a negative discrepancy. DIF: Comprehension REF: p. 316 2. A nursing unit’s census consists primarily of long-term residents with a high risk for falls. To meet new safety regulations, the nurse manager must plan to replace all 50 patient beds with new beds equipped with Fall Watch electronic sensors that will detect when patients get out of bed. The manager will be involved in which type of budgeting to replace the beds? a. Fiscal b. Labor c. Operational d. Capital ANS: D Capital budgets are concerned with major purchases such as equipment paid for over several years. DIF: Comprehension REF: pp. 320-321 3. A nurse manager is preparing a budget that does not base annual budgets on the revenue and expenditures of the prior year and has the advantage that outdated information is not integrated into the budget. The manager is using which budget method? a. Zero-based b. Incremental c. Labor d. Operational ANS: A The zero-based method is based on the assumption of no volume and no resources assigned; it essentially starts each new budget period at zero rather than building from past budgets. DIF: Comprehension REF: p. 321 4. A nurse on the unit is heard saying, “I am not going to document that I used four catheters to start that IV; it doesn’t matter anyway.” What action can help the staff nurse understand the financial budget goals of the unit? a. Have the nurse work in payroll for a week. b. Enroll the staff in continuing education units (CEU) for personal finance. c. Ask the nurse to represent the unit on the budget planning committee. d. Make the nurse responsible for monitoring all disposable equipment and supplies. ANS: C Participating on the committee will give the nurse ownership of the unit’s budget and will provide insight into the unit’s budgetary goals. DIF: Analysis REF: p. 326 5. In a large health care facility, the executive administrative leaders set the budget goals to decrease full-time equivalents by 3%, eliminate the cost of agency nurses, reduce lost revenue from lost supplies by 1%, and provide a 0.5% hourly salary incentive for working on a float unit when the assigned unit has a low census. Nurse managers meet with nursing administrators to design their unit budgets to meet these established goals. The budget approach that is being used is the approach. a. top-down b. participatory c. iterative d. incremental ANS: A Budget goals are established by administrators; unit managers do not contribute to goal setting, which is the primary principle of the top-down approach. DIF: Comprehension REF: pp. 321-322 6. Which component of budgeting might normally be addressed in the annual performance evaluation for a nurse manager? a. Including in the labor budget costs for overtime and benefits b. Managing variances in nursing overtime costs and supply usage c. Accurately predicting revenues on the basis of unit-of-service d. Providing qualitative analysis for variances in the capital budget ANS: B Nurse managers are often evaluated according to their success in managing nursing overtime costs and supply usage as reflected in the unit’s budget. DIF: Application REF: p. 319 7. A primary function of the budgeting process is to provide managers with an opportunity to: a. insist that salary increases for all nurses are included in the annual budget. b. discuss concerns about resource allocation with leaders of the organization who are capable of resolving issues. c. develop a mechanism for changing from zero-based budgeting to incremental budgeting. d. develop for all staff an educational program related to supply usage. ANS: B Coordination and communication are very important functions of budgeting that require many different groups within an organization to come together with organizational leaders to discuss the resources necessary to accomplish the goals of a business unit. DIF: Application REF: p. 318 8. Organizations measure the effectiveness of their budgets by examining actual revenues and expenditures versus: a. planned variances. b. incremental budgets. c. productivity metrics. d. expected performance. ANS: D Variance analysis is the process by which deviations from budgeted amounts are examined by comparing actual performance results against expected, or budgeted, performance. DIF: Comprehension REF: p. 323 9. When the nurse manager conducts a qualitative analysis of budget variances, he or she is: a. determining the percentage increase of supply usage from the last quarter to the current quarter. b. identifying the overall increase in the dollar amount of salaries paid for overtime. c. comparing productivity metrics across all nursing units in the facility. d. reconciling with current conditions the underlying assumptions on which the budget was based. ANS: D Qualitative analysis of the budget explains why current conditions are different than they were when the budget was developed; new conditions might include greater patient acuity or additional physicians with increased admissions. DIF: Application REF: p. 323 10. A nurse manager is working with the financial officer to develop the budget of the nursing unit for the next fiscal year. The nurse manager tells the financial officer that which of the following pieces of information will affect budget assumptions? a. The capital budget request for new emergency department equipment has been turned in for consideration by the hospital’s administrative team. b. The patient census likely will increase during the next fiscal year because two large physician groups have transferred their admission privileges to this hospital. c. The participatory budgeting approach instituted last year has been helpful in controlling supply costs. d. Zero-based budgeting will help the managers to be more efficient in establishing next year’s budget. ANS: B Budget assumptions are future predictors of performance and include the stability of the price of supplies, the salary range needed to recruit and retain quality employees, new services offered by competitors, and the variability of the patient census. DIF: Application REF: p. 322 11. The nurse manager meets with upper management to share strategic goals agreed upon by her staff for their individual nursing unit as the first step to begin budget negotiations. One strategic goal is that staff will have access to technology that will allow them to incorporate point-of-care devices for all RNs. The second goal is to improve RNs’ ability to recognize critical indicators that a patient’s health status is deteriorating through advanced health assessment skills. During the meeting the manager discusses the strategy options and selecting the one that works for the unit. Which type of budget development is used? a. Iterative b. Top-down c. Participatory d. Zero-based ANS: C In the participatory approach, the people responsible for achieving the budget goals are included in goal setting. DIF: Comprehension REF: p. 322 12. The nursing executive team met to review last year’s productivity metric to strategize for the upcoming year’s metric. The team wants to be certain the productivity metric shows productivity was: a. high. b. low. c. balanced. d. iterative. ANS: C A balanced productivity is desired because it encourages quality and safety while providing financial efficiency. DIF: Comprehension REF: p. 320 13. The nurse manager meets with upper administration and learns that the strategic plan for nursing is to have 80% BSN staff within the next 3 years. The nurse manager then built her budget to meet the organization’s strategic goal by providing tuition reimbursement and flexible work hours, which required some agency staffing. Which approach to budgeting is used? a. Iterative b. Top-down c. Participatory d. Zero-based ANS: A The iterative approach is a combination of the top-down and the participatory approach, with upper management defining strategic goals and then unit leaders developing their operating budgets to incorporate their individual goals in conjunction with the organization’s strategic goals. DIF: Comprehension REF: pp. 321-322 14. 14. The nurse managers of an organization are meeting with administration to plan the budget. The above graph shows last year’s 2011 expenditures, and the team will trend the upcoming budget knowing that, with the slowdown in the economy, patient census will be lower. Which budget approach is being used? a. Incremental b. Zero-based c. Productivity metrics d. Capital ANS: A The incremental approach is simply a forward trend of current or recent performance with adjustments for future growth or decline in revenues or expenses. DIF: Comprehension REF: p. 321 15. Nurses on a unit met with the nurse manager as part of participatory budgeting. They ask, “What exactly is the difference between fixed and variable costs? Understanding this will help us better understand the budgeting process.” The manager provides a definition and asks the staff to list types of direct and indirect cost. Which example would indicate a need for further teaching? a. Fixed costs would include accreditation fees. b. Fixed cost would include the cost for the automated medication-dispensing system. c. Variable cost would include the nurse manager’s salary. d. Variable cost would include the salary expense for registered nurses. ANS: D Variable costs would include the salary expense for registered nurses because their number can change. DIF: Analysis REF: p. 326 MULTIPLE RESPONSE 1. A nurse manager has calculated that providing 75 hours of direct nursing care per day requires that 120 hours must actually be worked by nursing staff. The manager is involved in: (select all that apply) a. developing the capital budget. b. applying productivity metric. c. monitoring the labor budget. d. incremental budgeting processes. e. addressing budget assumptions. ANS: B, C The nurse manager is determining the amount of work produced by calculating the actual number of nursing hours worked, which is productivity metric. Budgets use productivity metrics. DIF: Application REF: p. 320 2. A nurse has recently been appointed to the position of nurse manager. To become successful in managing the unit’s budget, the new nurse manager should: (select all that apply) a. read the financial policy manual to learn more about the organization’s budgeting process. b. allow the nursing administration office to manage the unit’s budget until he or she is able to complete an online financial management course. c. communicate regularly with a person in the hospital’s finance office about interpreting budget reports. d. pay attention to only the bottom-line numbers in budget reports rather than trying to understand each line in these reports. e. discuss the process of developing budgets with other members of the management team. ANS: A, C, E Knowing the organization’s financial policies is an important step toward understanding the organization’s budget process. Building a relationship with the finance office is fundamental to learning about the budgeting. As one participates more in the budget process, one’s understanding of the process and related responsibilities is enhanced. DIF: Application REF: p. 323, Box 18-3 COMPLETION 1. The financial plan required for the distribution of resources and expenses is a . ANS: budget A budget is an organization’s financial plan that expresses expected expenses (such as personnel and supplies) and anticipated revenues for products and services over a defined period, usually 12 months. DIF: Comprehension REF: p. 316 Chapter 19: Effective Communication and Conflict Resolution Cherry & Jacob: Contemporary Nursing: Issues, Trends, and Management, 7th Edition MULTIPLE CHOICE 1. A nurse is listening to a patient’s apical heart rate. The patient asks, “Is everything okay?” The nurse says nothing and shrugs her shoulders. The nurse is demonstrating: a. open communication. b. filtration. c. blocking. d. false assurance. ANS: C Blocking occurs when the nurse responds with noncommittal or generalized answers. DIF: Comprehension REF: p. 335 2. A teenage patient is using earphones to listen to hard rock music and is making gestures in rhythm to the music. The nurse assesses the amount of urine output in the Foley catheter and leaves the room. What communication technique is demonstrated in both of these situations? a. Blocking b. Filtration c. Empathy d. False assurance ANS: B Filtration is the unconscious exclusion of extraneous stimuli in communication. DIF: Comprehension REF: p. 331 3. In today’s world of fast, effective communication, what is the most commonly used means of societal communication? a. Facial expression b. Spoken word c. Written messages d. Electronic messaging ANS: B Verbal communication, which involves talking and listening, is the most common form of interpersonal communication. An important clue to verbal communication is the tone or inflection with which words are spoken and the general attitude used when speaking. DIF: Knowledge REF: p. 331 4. Which statement accurately describes communication? a. The components of communication are mutually exclusive. b. Communication is linear. c. Communication involves only the sender and the receiver; everything else is superficial. d. When the receiver becomes the sender, the subcomponent of communication that is in use is feedback. ANS: D Communication is a process that requires certain components, including a sender, a receiver, and a message. Effective communication is a dynamic process: With a response (feedback), the sender becomes the receiver, the receiver becomes the sender, and the message changes. DIF: Comprehension REF: p. 330, Figure 19-1 5. A licensed practical nurse (LPN) has been practicing for 25 years on a unit where a newly graduated RN with a bachelor’s degree is hired. Before the RN arrives on the unit, the LPN is heard saying, “She’ll try to tell everyone what to do because she makes more money. She’ll sit at the desk and let us do all the work.” This is an example of a(n): a. interpretation. b. context. c. precipitating event. d. preconceived idea. ANS: D Preconceived ideas are conceptions, opinions, or thoughts that the receiver has developed before having an encounter. Such ideas can dramatically affect the receiver’s acceptance and understanding of the message. DIF: Application REF: p. 330 6. A new mother is experiencing pain after delivering an infant with Down syndrome. The staff nurse states, “I don’t think she is really hurting. Let the next shift give the pain medication.” The team leader notices the staff nurse looks agitated and anxious and asks about any concerns in providing care to this new mom. The staff nurse admits having a stillborn infant with Down syndrome. This is an example of which component of communication? a. Personal perception b. Past experiences c. Filtration d. Preconceived idea ANS: B With past experiences that include a variety of positive, neutral, and negative events, the influence that these experiences can and will have on communication may be positive, neutral, or negative. The importance of recognizing that any reaction from the receiver may be biased by previous experience cannot be overstated. DIF: Application REF: p. 331 7. A nurse gives Dilantin intravenously with lactated Ringer’s solution containing multivitamins. The drug precipitates and obstructs the only existing line. When the team leader informs the nurse that these drugs cannot be mixed, the nurse states, “Everyone just pushes the medicine slowly. No one checks for compatibility. There isn’t even a compatibility chart on the unit.” Which type of logical fallacy has influenced the nurse? a. Ad hominem abusive b. Appeal to common practice c. Appeal to emotion d. Appeal to tradition ANS: B An appeal to common practice occurs when the argument is made that something is okay because most people do it. DIF: Application REF: p. 336 8. An RN is consistently late to work, causing reassignment of patient care and the need for repeated shift reports. The nurse, who receives a warning for repeated tardiness, states, “My husband left me, I have no car, no family close by, and the bus is always late, which makes me late. The nurse manager doesn’t care how hard I try to get here, and I am raising a child by myself.” The nurse is using which type of logical fallacy? a. Appeal to emotion b. Appeal to tradition c. Hasty generalization d. Confusing cause and effect ANS: A An appeal to emotion is an attempt to manipulate other people’s emotions for the purpose of avoiding the real issue. DIF: Application REF: p. 336 9. The new director of nurses has instituted “walking rounds” on all nursing units, rather than the usual taped shift reports. A veteran nurse exclaims, “She doesn’t know how we do things here!” The nurse is demonstrating: a. appeal to emotion. b. appeal to tradition. c. red herring. d. straw man. ANS: B An appeal to tradition is the argument that doing things a particular way is best because they’ve always been done that way. DIF: Application REF: p. 336 10. A male nurse hired to work in the emergency department is observed throwing a contaminated needle into the trash can. The team leader reprimands the nurse for not appropriately disposing of sharps. The nurse states, “You don’t care that I threw the needle in the trash. You just want an all-female staff,” putting the team leader in a defensive position. This communication technique is known as: a. straw man. b. red herring. c. slippery slope. d. confusing cause and effect. ANS: B A red herring is the introduction of an irrelevant topic for the purpose of diverting attention away from the real issue. DIF: Application REF: pp. 336-337 11. A nurse who was recently certified in chemotherapy administration fails to check compatibility of phenytoin (Dilantin) before injecting into a continuous infusion of D5W leading to occlusion of the line. Which statement by the nurse demonstrates a red herring? a. The nurse is upset and states, “I am sure I have injected this before without a problem” and the supervisor interprets this to mean the nurse often take shortcuts. b. The nurse states, “You are just upset because I am certified in chemotherapy administration and you are not.” c. “The nurse who started the IV didn’t get a blood return but determined the IV was the patient’s—that is the problem.” d. “This drug always occludes the line because it is so viscous.” ANS: B The nurse diverts attention away from the issue of not checking compatibility to introduce an irrelevant topic of chemotherapy administration certification which is not related to this situation. DIF: Application REF: pp. 336-337 12. During a health history interview, the nurse listens to a patient relating the precipitating events that led to the onset of chest pain. She focuses her attention on the patient, makes eye contact, and acknowledges what the patient has to say. The nurse is exhibiting: a. assertive communication. b. active listening. c. empathy. d. passive communication. ANS: B In active listening a number of techniques can be used by the receiver to enhance the ability to listen; these include (1) providing undivided attention, (2) giving feedback (rephrasing), (3) making eye contact, (4) noting nonverbal messages (body language), and (5) finishing listening before one begins to speak. DIF: Comprehension REF: p. 337 13. The nurse is demonstrating active listening when: a. while assessing the patient’s vital signs, the nurse records the data and states, “You are improving, your vital signs are normal.” b. eye contact is maintained while focusing on the patient as the patient describes the current pain level and location. c. he or she states, “I know how you feel, I recently lost my father and I am still hurting.” d. cultural values are in opposition to the patient but shares that “I agree with your decision to use herbs rather than the prescribed medications.” ANS: B The behavior demonstrates active listening. A number of techniques can be used by the receiver to enhance the ability to listen; these include (1) providing undivided attention, (2) giving feedback (rephrasing), (3) making eye contact, (4) noting nonverbal messages (body language), and (5) finishing listening before one begins to speak. DIF: Application REF: p. 337 14. An older adult is unable to reach the telephone and is found dead at home several hours later. The son of the deceased person arrives at the hospital and asks, “Can I just please stay and hold my dad’s hand? He was so afraid of dying alone.” Which response by the nurse shows empathy? a. “You are just too late for that. Where were you when he needed you?” b. “Did you ever consider purchasing a cell phone for your dad to prevent this from happening?” c. “I’ll close the door so you can spend time with your dad. I will check back in a few minutes.” d. “I lost my dad last year. He died alone. He was a policeman. I am just like you. Let me stay here and console you.” ANS: C Empathy is demonstrated by the ability to mentally place oneself in another person’s situation to better understand the person and to share the emotions or feelings of the person. DIF: Application REF: p. 334 15. A nurse is preparing an exercise program as part of a health promotion program for older adults with osteoporosis. Which question would retrieve the most valuable information about health practices? a. “Do you exercise?” b. “Do you like to exercise?” c. “When do you exercise?” d. “What exercise practices do you participate in?” ANS: D “What exercise practices do you participate in?” is an open-ended question or statement that requires more information than just yes or no. This type of question augments the gathering of enough facts to build a more complete picture of the circumstances. DIF: Analysis REF: p. 334 16. Which component of an e-mail shown below would be both effective and concise? a. Subject: A short concise subject line: Meeting. b. Body: I would like you to answer these questions before the next meeting: Where would you like to meet? Do you want all the staff to attend? Can we serve refreshments? What is one goal for our unit? c. Body: Dear Staff, As you know, each department must reduce staff by 2%. We will need to discuss how to inform unlicensed staff about the downsizing efforts of the hospital. d. Body: The next staff meeting is scheduled for Tuesday, January 19, at 5 PM in the first floor auditorium. Please send items for the agenda. Sally Smith, MSN, RN, [email protected] or ext. 5582. ANS: D This provides a message that is concise and accurate with a clearly conveyed message for the reader and contact information from the sender, all of which are important components of effective e-mail communication. DIF: Analysis REF: p. 339 17. During height and weight assessments at a school’s health fair, a child admits to drinking a cup of coffee with his mother every morning, and another child reports enjoying a morning cup of coffee on the commute to school. These two children are both below average on the height chart, and the nurse states, “Drinking coffee stunts a child’s growth.” This logical fallacy is referred to as: a. appeal to common practice. b. confusing cause and effect. c. ad hominem abusive. d. red herring. ANS: B Cause and effect are confused when one assumes that a particular event must cause another just because the two events often occur together. DIF: Application REF: p. 336 18. A patient’s spouse was just diagnosed with lung cancer although there was no history of tobacco use. The spouse states, “I am so mad. How can you get cancer without smoking?” Which statement by the nurse represents empathy? a. “Research is identifying many risk factors for cancer besides smoking.” b. “I understand how you could feel angry about the diagnosis.” c. “He is still a good husband.” d. “Why do you think he got cancer?” ANS: B The nurse is placing herself in the wife’s position and sharing her emotions. DIF: Application REF: p. 334 19. A nurse wants to apply open communication to obtain a thorough history and to determine cognitive function. Which question represents the use of open communication? a. Is today Wednesday? b. Do you know what day it is? c. Tell me what day of the week today is. d. Do you know what the first day of the week is? ANS: C The patient must be able to name the day of the week rather than use answer yes or no. DIF: Application REF: p. 334 20. The nurse caring for a patient states, “Your blood pressure is dangerously high. Are you taking antihypertensive medicine?” The patient states, “I can’t afford my medicine. I have no insurance.” The nurse states “I feel really sorry for that patient. I wish it wasn’t against policy to give her money.” The nurse wants to help and places a note on Facebook that any donations would be appreciated to help a waitress who works at the cafeteria next door to the hospital buy her medications. The nurse posts that “She was so sick last evening when she came to the ED. I can’t believe they don’t provide insurance. I can’t give her money but you all can help.” This nurse: a. is showing empathy and as long as she lets the patient know the money is not from her, she is not violating any social media guidelines. b. is at risk for HIPAA violations. c. has properly followed policy and protected the patient by not using her name. d. is demonstrating the logical fallacy of slippery slope. ANS: B The National Council State Board of Nursing’s policy on social media prohibits posting of patient information on social media sites. This patient could be identified by knowing where she works and the fact that she was seen in the ED the day before. DIF: Application REF: p. 344 21. A group of nurses are meeting to decide how to staff the upcoming holidays. Each of the four members freely expresses thoughts about fair staffing but is willing to listen to other thoughts and reconsider their first recommendations. The nurses are avoiding conflict and supporting professional communication through: a. empathy. b. positiveness. c. supportiveness. d. accommodation. ANS: C Supportive communication occurs when each person’s opinion/position is valued and each participant has the freedom to express a position but is willing to change that opinion/position. DIF: Comprehension REF: p. 346 22. A nurse is asked to “float” to another area where the patients require total care. The nurse smiles, picks up her stethoscope, and says, “I’ll come back and eat lunch with everyone here.” When she enters the elevator she hits the wall and mutters, “Always me. Don’t I have any rights”? The nurse is demonstrating which communication style? a. Assertive b. Aggressive c. Passive d. Passive-aggressive ANS: D Passive-aggressive communication is represented by incongruent actions—the nurse shows friendly gestures by smiling and demonstrating she wants to have lunch with the staff on the original unit; however, she shows her anger by hitting the wall and muttering. DIF: Comprehension REF: p. 341 23. A nurse is overhead saying, “I don’t mind working during the election and holiday. My parents are divorced, money is tight, and honestly I don’t trust any politicians anyway. I plan to take a few weeks off next month.” She works independently to research strategies to improve patient-centered care for the large number of immigrants that arrived in the area and then works with the team to share ideas. She recommends, “Let’s think the suggestions over and come back together next week.” This nurse’s communication style is consistent with which generation? a. Baby Boomers b. Traditionalist c. Generation X d. Millenniums ANS: C Generation X individuals grew up in when there was a high rate of divorces; they tend to be more cynical and value work-life balance and teamwork. Holidays are often associated with family gatherings which may have been absent in this generation’s family; belief that ALL politicians are untrustworthy may be viewed as cynical. The need to take time to form suggestions to save time and come together to reach team decision are reflective of this generation. DIF: Comprehension REF: p. 343 COMPLETION 1. The healthiest form of communication is the style. ANS: assertive Assertive communicators are honest and direct while valuing and respecting other individuals’ views and seeking a win-win solution without the use of manipulation or game-playing. DIF: Knowledge REF: pp. 340-341 Chapter 20: Effective Delegation and Supervision Cherry & Jacob: Contemporary Nursing: Issues, Trends, and Management, 7th Edition MULTIPLE CHOICE 1. The task of completing and signing the initial assessment on a newly admitted patient who is about to undergo minimally invasive procedures on an outpatient basis can be delegated to: a. the registered nurse (RN). b. the licensed practical/vocational nurse (LPN/LVN). c. unlicensed assistive personnel (UAP). d. all levels of staff, because the information is about the past and cannot change. ANS: A Only the RN can perform and sign the admission assessment, although some components such as monitoring vital signs may be delegated. DIF: Comprehension REF: p. 357 2. An RN recently relocated to another region of the country and immediately assumed the role of charge nurse. When determining the appropriate person to whom to delegate, the RN knows that: a. the role of the LPN/LVN is the same from state to state. b. the LPN/LVN can be taught to perform all the duties of an RN if approved by the employer and if additional on-the-job training is provided. c. he or she must review the state’s nurse practice act for LPN/LVNs, because each state defines the role and scope of practice of the LPN/LVN. d. The Joint Commission has certified and established roles for the LPN/LVN. ANS: C The scope of practice of the LPN/LVN varies significantly from state to state; RNs should know the LPN/LVN nurse practice act in the state in which they practice and should understand the legal scope of practice of the LPN/LVN. DIF: Comprehension REF: p. 354 3. Which task is most likely to be considered in a state’s practice act as appropriate to delegate to a LPN/LVN if the patient’s condition is stable and competence in the task has been established? a. Administer an enema for an elective surgery patient. b. Administer an antiarrhythmic medication IV while interpreting the patient’s rhythm on the cardiac monitor. c. Develop a plan of care for a stable patient admitted for observation after a head injury. d. Teach a patient how to instill eye drops for glaucoma. ANS: A The RN who is delegating must consider the following: (1) the delegatee’s current workload and the complexity of the task, (2) whether the staff member is familiar with the patient population and with the task to be performed, and (3) whether the RN is able to provide the appropriate level of supervision. The delegation decision-making tree would also support delegation of this task. DIF: Analysis REF: pp. 357-358, Box 20-3 4. A patient is admitted with hypotension, shortness of breath, flushing, and hives. All levels of staff have been trained to assess vital signs. Given budget restrictions and proper delegation rules, to which care provider would the RN delegate the task of obtaining the initial blood pressure reading? a. RN b. LPN/LVN c. Unlicensed assistive personnel (UAP) d. Use the blood pressure obtained in the ambulance, because it was assessed via electronic monitoring. ANS: A The patient’s condition is not stable; therefore, the skills of an RN are required. DIF: Application REF: pp. 357-358, Box 20-3 5. A nurse is delegating to the newly hired nursing unlicensed assistive personnel (UAP) the task of assisting with oral hygiene, knowing that this assignment “does not require decisions based on the nursing process.” The nurse is correctly using which of the five rights of delegation? a. Supervision b. Communication c. Person d. Circumstance ANS: D Right circumstance involves the delegation of tasks that do not require independent nursing judgments. DIF: Comprehension REF: p. 360 6. A student nurse is concerned about delegation practices and wonders why hospitals employ unlicensed assistive personnel (UAP) and LPN/LVNs. The student nurse refers to the National Council of State Boards of Nursing and learns that the role of these personnel is to: a. supplement the staffing pattern when an RN is not available. b. aid the RN by performing appropriately delegated care tasks. c. replace the RN when the health care facility provides long-term care. d. provide patient teaching, allowing more direct care to be provided by the RN. ANS: B The UAP and LPN/LVN can increase productivity of the RN by performing those tasks that fall within their scope of practice. DIF: Comprehension REF: p. 353 7. An RN delegates to the unlicensed assistive personnel (UAP) the task of performing blood pressure checks for a group of patients on a nursing unit. The UAP accepts the task and is responsible for: a. delegating the task to another UAP if he or she does not have the time or skill to complete the task. b. keeping the RN informed of any abnormal blood pressure readings. c. calling the physician when the patient’s vital signs are not within established parameters. d. informing the dietary department to initiate a low-sodium diet for patients who are hypertensive. ANS: B After accepting the assignment, the UAP is responsible for completing the task and reporting any patient concerns to the RN. DIF: Application REF: p. 354 8. Which task is appropriate for the RN to delegate to the unlicensed assistive personnel (UAP) provided the delegatee has had experience and training? a. Evaluate the ability of a patient to swallow ice after a gastroscopy. b. Assist a patient who is postoperative hip replacement to ambulate with a walker for the first time. c. Change the disposable tracheotomy cannula for a new postoperative tracheotomy patient if secretions are thick and tenacious. d. Obtain a sterile urine sample from a patient with a Foley catheter that is connected to a closed drainage system. ANS: D Obtaining a sterile urine sample from a patient with a Foley catheter that is connected to a closed drainage system is not an invasive procedure, and risk to the patient is minimal, making the task appropriate for delegation. DIF: Analysis REF: pp. 357-358, Box 20-3 9. An LPN/LVN has transferred to a nursing unit and arrives for the first day. The RN checks with the LPN/LVN often throughout the shift to provide support and determine if assistance is needed. The RN is providing which level of supervision? a. There is no supervision, because at times the LPN/LVN is not with the RN. b. Periodic inspection is being used. Because the LPN/LVN is licensed, the RN is relieved of the need to evaluate care. c. Continual supervision is being provided until the RN determines competency. d. Initial supervision is being provided because this is the LPN/LVN’s first day on the unit. ANS: C This level of supervision is required when the working relationship is new, the task is complex, or the delegatee is inexperienced or has not demonstrated an acceptable level of competence. DIF: Comprehension REF: p. 362 10. An RN is counseled by the nurse manager regarding inappropriate delegation when the: a. RN instructs the nursing assistant to greet ambulatory surgery patients and show them to their rooms. b. nursing assistant informs the RN that she has not been trained to collect a sputum specimen and the RN states, “I will show you this time and you can show me the next time.” c. RN assigns the float LPN/LVN the task of completing a plan of care for a stable patient who was admitted for routine replacement of a feeding tube. d. LPN/LVN who has demonstrated competence is asked to perform a dressing change for a patient before she is discharged home. ANS: C Only an RN can initiate and complete a new plan of care; this does not fall within the scope of practice of the LPN/LVN. The RN has violated one of the five rights of delegation. DIF: Application REF: p. 357 11. An RN delegates to an experienced LPN/LVN the task of administering oral medications to a group of patients. The LPN/LVN accepts the assignment, and the RN knows that the LPN/LVN has had the training and has acquired the skills needed to complete the task. The RN then observes the LPN/LVN recording a patient’s medication administration just before entering the patient’s room. The priority intervention by the RN is to: a. check the patient’s drug packages to ensure that the correct drugs were given. b. stop the LPN/LVN immediately and discuss the possible consequences of his actions in a nonjudgmental manner. c. contact the nurse manager and ask that the LPN/LVN’s license be suspended. d. call the pharmacy and ask for replacement medications for the patients. ANS: B The LPN/LVN has the competency but violated one of the rights of medication administration and is practicing unsafe care. The RN’s responsibility requires that he or she intervene and identify concerns with the LPN/LVN. DIF: Analysis REF: p. 363 12. Which statement related to delegation is correct? a. The practice of unlicensed assistive personnel (UAP) is defined in the nurse practice act. b. Nursing practice can be delegated only when the LPN/LVN and UAP have received adequate training. c. Supervision is not required when routine tasks are delegated to a competent individual. d. The RN must be knowledgeable about the laws and regulations that govern nursing practice, as well as those that have no clearly defined parameters, such as for UAP. ANS: D Accountability remains with the RN, and he or she is responsible for knowing what tasks can be delegated and what is defined as nursing practice. DIF: Knowledge REF: p. 354 13. During orientation, an RN learns that LPN/LVNs in the facility receive additional training to perform some tasks such as hanging continuously infusing intravenous fluids that have no additives. It is important for the RN to understand that: a. the health care facility can override the state practice act by having all LPN/LVNs and unlicensed assistive personnel (UAP) participate in on-site training. b. LPN/LVNs are licensed, and accountability for their own practice rests with each LPN/LVN. c. UAPs cannot be held responsible for their own actions or inactions. d. the nurse practice act and state regulations related to delegation override the organization’s policies. ANS: D The state’s nurse practice act is the deciding factor regarding what can legally be delegated. DIF: Comprehension REF: p. 355 14. A nurse moves from California to Arkansas and due to having 20 years of experience as a registered nurse is immediately placed in charge of the telemetry unit. The staffing consists of LPNs and two unlicensed assistive personnel. The RN is unsure of the scope of practice of the LPNs and reviews the nurse practice act for Arkansas, which lacks clarity on some tasks. The RN should: a. query the state nursing association to determine their stance on the role of LPNs. b. ask the LPNs on the unit to list what tasks they routinely performed. c. contact the state board of nursing to determine legal scope of practice for LPNs. d. refer to California’s nurse practice act because the scope of LPNs/LVNs is consistent across the United States. ANS: C If the nurse practice act lacks clarity, the state board of nursing can provide guidance. DIF: Application REF: pp. 354-355 15. An RN makes the following assignments at the beginning of the shift. Which assignment would be considered high-risk delegation? a. A novice RN is assigned a patient with diabetes mellitus requiring mixing of regular and NPH insulin. b. An LPN is assigned an older adult with pneumonia and who requires dressing changes on a foot wound. c. An unlicensed assistive person is assigned the task of assisting a patient with late stages of Huntington’s disease to ambulate a short distance in the hallway. d. A float RN from the oncology unit is assigned a patient with a white blood cell count of 4000 mm3. ANS: C Risk of falling is great in later stages of Huntington’s disease due to chorea movements. DIF: Application REF: pp. 359-360 16. The RN instructs the LPN to “Give an enema to the patient in room 327 who is being discharged but is complaining of being constipated. Then be sure to document on the medication administration record when given.” Which of the five rights was missing in this situation? The right of: a. direction and communication. b. task. c. person. d. circumstances. ANS: A The directions were not clear. The RN did not specify which type of enema to give and what outcome to expect. And the RN gave no instructions related to reporting back. DIF: Application REF: p. 360 17. Which of the following situations would be appropriate for the supervisory level of initial direction and/or periodic inspection? a. Experienced RNs work together to provide care for a group of patients newly diagnosed with meningitis. b. The RN assigns the LPN tasks within her scope of practice and checks back during the shift to ensure the tasks are completed correctly. c. A new graduate nurse is assigned care to a male patient with a hematocrit of 11.0 g of hemoglobin per deciliter and is receiving a blood transfusion. The charge nurse checks on the patient status every 15 to 30 minutes and asks the graduate to explain “next steps.” d. No supervision is necessary since both are registered nurses. ANS: B When a working relationship is established and competencies of the delegate established, the delegator may check in during intermittently during the shift. DIF: Application REF: p. 361 18. A registered nurse (RN) is assigned as charge nurse for the first time. She knows to consult the state board of nursing to determine scope of practice for licensed practical nurses (LPN) and unlicensed assistive personnel (UAP). She also realizes there are common policies which exist in most state practice acts that include: a. the RN is held accountable for the decision to delegate, but responsibility rests only with the delegatee. b. the RN may only delegate tasks that are not in the scope of practice of the LPN if the delegatee is certain they are competent to perform the task. c. since the LPN is licensed, they practice professional nursing. d. to determine what tasks can be safely delegated, the RN must first assess the patient. ANS: D The stability of the patient must be determined prior to delegation. Even routine tasks such as taking vital signs that are often delegated may need to be performed by the RN when the patient’s condition is critical. DIF: Comprehension REF: pp. 354-355, Box 20-1 19. A nurse is concerned about the risk of delegating tasks to licensed practical nurses and unlicensed assistive personnel. What is the best way for the nurse to determine competency of an inexperienced delegatee? a. Actually observe the delegatee perform the assigned task. b. Ask the delegatee how many times he/she has performed the task. c. Ask the patient if the care provided was satisfactory. d. Ask other nurses if they feel the delegatee is competent. ANS: A The best way for the nurse to determine the competency of LPNs or UAPs is to observe them perform the task. DIF: Application REF: p. 361 20. Care delivery using the team-based approach is used on a telemetry nursing unit. The team consists of one registered nurse (RN), two licensed practical nurses (LPNs), and one unlicensed assistive personnel (UAP). Staff have been charged to improve quality of care while ensuring cost containment. Which assignments would meet both criteria? a. The RN administers all medications to all patients. b. The LPN performs sterile dressings and IV tubing changes on all central lines. c. The experienced UAP places telemetry electrodes and attaches to cardiac monitor. d. The RN administers an enema to a stable patient who has an order “administer fleet enema PRN when no bowel movement in 2 days.” ANS: C The UAP, when properly trained, can place patients on telemetry. This meets quality and cost containment goals because the LPN and RN have higher salaries. DIF: Application REF: pp. 353-354 MULTIPLE RESPONSE 1. Which statement made by an RN regarding delegation indicates the need for additional teaching? (select all that apply) a. Unlicensed assistive personnel (UAP) can assess vital signs during the first 5 minutes for a patient who is receiving a blood transfusion because a reaction at this time is unlikely. b. An LPN/LVN can administer a PPD (tuberculin skin test) if there is no history of a positive PPD. c. When dopamine is ordered continuously, the LPN/LVN can administer dopamine at a low dose for the purpose of increasing renal perfusion. d. UAPs can transfer a patient who is being discharged home from the wheelchair to the bed if they have received training and demonstrated competency. e. Responsibility can be delegated to the UAP, but the delegator retains accountability. ANS: A, B, C The statement “UAPs can assess vital signs during the first 5 minutes for a patient who is receiving a blood transfusion because a reaction at this time is unlikely” indicates the need for further teaching because the patient is at highest risk of a reaction during the first few minutes of a blood transfusion; thus the assessment skills of an RN are required. The statement “an LPN/LVN can administer a PPD (tuberculin skin test) if there is no history of a positive PPD” indicates the need for further teaching because administration of intradermal medication requires the skill of an RN. Dopamine is a vasoactive drug that can have a profound effect on a patient’s blood pressure and cardiac output; administration requires the assessment and evaluation skills of an RN. DIF: Analysis REF: pp. 357-358, Box 20-3 2. A nursing administrator who is considering the feasibility of an all-RN staff reviews the report, Keeping Patients Safe: Transforming the Work Environment of Nurses (2003) and determines that RNs: (select all that apply) a. are more costly and less efficient than LPNs. b. have little or no effect by being proactive but instead are reactive to patient care errors. c. have a positive effect on patient outcomes when managing patient care. d. are effective overseers of patients’ overall health condition. e. lack the training to be effective delegators. ANS: C, D RNs are effective at coordinating care that results in improved patient outcomes. RNs are valuable monitors of a patient’s health status—a practice that results in improved patient outcomes. DIF: Comprehension REF: p. 353 3. Which functions can be delegated only to another RN with appropriate experience and training? (select all that apply) a. Assessment of skin integrity on third day of hospitalization b. Evaluation of patient teaching related to turn, cough, and deep breathing exercises c. Nursing judgment related to withholding medication based on vital signs d. RNs do not delegate to other RNs, they delegate only to licensed practical nurses or unlicensed assistive personnel e. Formulation of nursing diagnosis “potential for fall” ANS: A, B, C, E Activities like assessing skin integrity—which include the core of the nursing process and require specialized knowledge, judgment, and/or skill—can be delegated only to another RN. Activities like evaluating patient teaching—which include the core of the nursing process and require specialized knowledge, judgment, and/or skill—can be delegated only to another RN. Activities like deciding to withhold medication based on vital signs—which include the core of the nursing process and require specialized knowledge, judgment, and/or skill—can be delegated only to another RN. Activities like formulating a nursing diagnosis—which include the core of the nursing process and require specialized knowledge, judgment, and/or skill—can be delegated only to another RN. DIF: Application REF: pp. 361-362 Chapter 21: Staffing and Nursing Care Delivery Models Cherry & Jacob: Contemporary Nursing: Issues, Trends, and Management, 7th Edition MULTIPLE CHOICE 1. Accrediting agencies such as The Joint Commission address staffing by: a. imposing maximum staffing levels. b. requiring a specific staff mix. c. stipulating nurse-patient ratios. d. looking for evidence that patients receive satisfactory care. ANS: D Accrediting agencies do not address minimum staffing levels; however, they do look for evidence that patients receive adequate care, and this can occur only with adequate staffing. DIF: Comprehension REF: p. 369 2. Customer satisfaction is primarily based on: a. access to modern, up-to-date facilities. b. availability of an extensive menu selection. c. personal interactions with employees. d. having to undergo fewer invasive procedures. ANS: C Interactions between employees and patients/families actually affect clinical outcomes, functional status, and even physiologic measures of health. DIF: Knowledge REF: p. 369 3. rNurses on a unit provide personal hygiene, administer medications, educate the patient and family about treatments, and provide emotional support. These nurses provide patient care based on which nursing delivery system? a. Total patient care b. Partnership nursing c. Team nursing d. Functional nursing ANS: A In total patient care nurses provide all aspects of patient care. DIF: Comprehension REF: p. 370 4. A hospital converts to a system of care delivery in which RNs, LPNs, and unlicensed assistive personnel (UAP) are responsible for implementing a specific task, such as medication administration or personal hygiene, for the entire nursing unit. This type of delivery system is: a. total patient care. b. functional nursing. c. team nursing. d. primary nursing. ANS: B In functional nursing members of the team are assigned specific tasks such as assessment or medication administration. DIF: Comprehension REF: pp. 370-371 5. The nurse who is responsible for following the patient from admission through discharge or resolution of illness while working with a broad range of health care providers is called a: a. nurse manager. b. case manager. c. coordinator of patient-centered care delivery. d. team leader in team nursing care delivery. ANS: B The case manager, in collaboration with an interdisciplinary team, oversees the use of health care services by clients throughout a course of illness. DIF: Comprehension REF: p. 373 6. A patient is admitted with coronary artery disease and is scheduled for coronary artery bypass grafting (CABG). According to the clinical pathway the patient should be extubated and discharged from critical care the day after surgery. During surgery the patient’s oxygen saturation decreased drastically as a result of chronic tobacco abuse. Subsequently, the patient remained on the ventilator an additional 2 days postoperatively. According to the clinical practice guideline for CABG, this situation represents a: a. patient outcome. b. variance. c. goal. d. standard. ANS: B A variance is a deviation from the planned path. DIF: Comprehension REF: p. 375 7. A patient is admitted with pneumonia. The case manager refers to a plan of care that specifically identifies dates when supplemental oxygen should be discontinued, positive-pressure ventilation with bronchodilators should be changed to self-administered inhalers, and antibiotics should be changed from intravenous to oral treatment, on the basis of assessment findings. This plan of care is referred to as a: a. patient classification system. b. clinical pathway. c. patient-centered plan of care. d. diagnosis-related group (DRG). ANS: B A clinical pathway is a plan that specifies the timing and sequencing of major patient care activities and interventions by the interdisciplinary team for a particular diagnosis, procedure, or health condition. DIF: Application REF: pp. 374-375 8. The nurse manager determines that four RNs, five LPN/LVNs, and two unlicensed assistive personnel (UAP) are required per shift to meet the needs of the patient population on the unit, according to acuity and census. The nurse manager is concerned with: a. assignments. b. staffing. c. output. d. productivity. ANS: B Staffing is the activity of determining that an adequate number and mix of health care team members are available to provide safe, high-quality patient care. DIF: Comprehension REF: p. 367 9. A nurse manager is mentoring a novice nurse manager in determining staffing needs. The mentor explains, “We must determine the acuity level of the patient by: a. assessing patient satisfaction with nursing care.” b. quantifying the amount and intensity of care required.” c. examining the skill mix and educational preparation of the staff.” d. determining the number of hospital days required by the patients.” ANS: B Patient acuity is measured by determining the amount and intensity of care required. DIF: Comprehension REF: p. 367 10. The nurse manager is planning staffing levels and realizes that the first step is to: a. know the intensity of care needed by patients according to physical and psychosocial factors. b. examine the educational level of the staff. c. assess the skill level of caregivers. d. review the budget to determine the financial consequences of past staffing patterns. ANS: A The nurse manager must determine the number and mix of health care providers according to the wide range of care requirements of individual patients. DIF: Application REF: p. 367 11. A hospital is concerned with nurse retention and realizes that job satisfaction is a major influence. To enhance employee satisfaction related to staffing, the management team: a. negotiates for additional agency nurses. b. hires more part-time employees. c. includes participatory management into staffing decisions. d. uses “float” nurses to cover vacancies. ANS: C Staffing methods that include staff participation and enhance staff autonomy have been demonstrated to play a major part in ensuring employee satisfaction. DIF: Application REF: p. 368 12. A patient is admitted for a hysterectomy, and the RN develops and implements the plan of care but also delegates to the LPN/LVN the responsibility of administering oral medications. While off duty, this RN receives a call requesting a change in the plan of care because the patient has developed deep vein thrombosis. The nurse who originally planned the care is practicing which type of nursing care delivery? a. Modular b. Primary c. Team d. Functional ANS: B The primary nurse assumes 24-hour responsibility for planning, directing, and evaluating the patient’s care from admission through discharge but may delegate or provide primary care during the shift when present. DIF: Comprehension REF: p. 372 13. When deciding which staffing option to use on a nursing unit that will open soon, the manager realizes that: a. continuity of care is enhanced and errors are reduced when nurses provide care over longer shifts and consecutive workdays, such as 12-hour shifts on 3 consecutive days per week. b. the use of part-time nurses provides the variability needed to meet diverse patient needs. c. satisfaction of the staff equates to satisfaction of patients. d. nurses provide the same level of care, regardless of the work environment. ANS: C High nurse satisfaction is generally equated with high patient satisfaction and positive patient outcomes. DIF: Comprehension REF: p. 368 14. A task force is considering factors that contribute to high-quality safe staffing. Which statement reflects an understanding of the American Nurses Association’s (ANA) recommendations? a. Because patient needs remain constant on a daily shift, staffing needs at the beginning of the shift should be sufficient to provide safe, high-quality care. b. Staffing should allow time for the RN to apply the nursing process so decisions result in high-quality, safe patient outcomes. c. Patient acuity levels affect staffing by increasing the need for unlicensed personnel to provide routine basic care rather than increasing RNs in staff mix. d. RN staffing is not cost-effective; thus is it important for staffing models to limit the number of RNs assigned per shift. ANS: B The ANA recommends that nurses have time to exercise professional judgment. DIF: Analysis REF: p. 367 15. A nursing unit is comparing team nursing to the partnership model and finds that: a. with the partnership model, an RN does not have to be part of the mix. b. leadership abilities of the RN is a major determinant of effectiveness of care for both models. c. the RN teaches the LPN/LVN or unlicensed assistive personnel (UAP) how to apply the nursing process in team nursing. d. with team nursing the RN cares for the patient while the team members work with the family or significant others. ANS: B The RN leads regardless of whether partnership model or team nursing is practiced. DIF: Comprehension REF: p. 371 |p. 372 16. A nurse groups patients with criteria such as “high risk for falls,” “infection protocols,” and “special communication needs” to determine the mix and number of staff needed on a telemetry unit. The nurse is using: a. a patient classification system to determine safe staffing levels. b. diagnostic-related groups for Medicare billing. c. case management to coordinate care. d. clinical pathways to determine care. ANS: A Patient classification systems group patients according to care needs to determine safe staffing levels. DIF: Comprehension REF: p. 367 17. A nurse makes patient care assignments as follows: RN1 has rooms 200-210; RN2 has rooms 211-221; RN3 has rooms 222-232. The two unlicensed assistive personnel have half the rooms, with one assigned to 200-215 and the second to 216-232. The care delivery model used in this situation is: a. team. b. primary. c. partnership. d. modular. ANS: D Modular (or geographic) assignments are based on a geographic location in the nursing unit. DIF: Comprehension REF: p. 372 18. A patient has decided to stop hemodialysis because his renal failure progresses and he wishes to spend more time with family. Palliative care will continue, and the approach will be discussed with the patient and family as needed and at change of shift. The care delivery model in this situation is termed: a. partnership. b. patient-centered. c. case management. d. total patient care. ANS: B Patient-centered care models entail the health care team partnering with the patient and family to ensure that patients’ wants, needs, and preferences are the priority while allowing the patient and family to participate in decisions and educational needs. DIF: Comprehension REF: p. 373 19. A nurse plans care knowing when specific recovery milestones are expected. The nurse is providing care via: a. patient classification systems. b. clinical pathways. c. functional nursing. d. case management. ANS: B Clinical pathways plans patient care activities and interprofessional interventions and desired patient outcomes within a specified time period for a particular diagnosis or health condition. DIF: Comprehension REF: p. 366 20. 20. An orthopedic unit is considering different types of care delivery models and staff have an opportunity to ask questions about how the models differ. The nurse manager provides an overview and uses the above visual to demonstrate which model of care delivery? a. Team b. Partnership c. Primary d. Functional ANS: D Functional care delivery models assign tasks to each provider. In the above visual, the LPN is responsible for oral medication administration, the unlicensed assistive personnel provide hygiene, and the RN is assigned to task that require the nursing process. DIF: Application REF: pp. 370-371 MULTIPLE RESPONSE 1. While participating in a task force to proactively plan for nursing care delivery over the next 20 years, a nurse learns that dramatic changes will occur as a result of: (select all that apply.) a. the increase in the number of minimally invasive procedures being performed for disease treatment. b. care provided for patients over an extended period in acute care settings. c. the reduction in the number of nurses and other health care professionals who are available to provide care. d. the widespread illiteracy and decreased self-efficacy of the aging patient population. e. the need to focus on social and environmental influences, educational level, and individual characteristics and values of the patient. f. the devaluing of nursing as a means of improving patient outcomes. ANS: A, C, E Invasive surgical procedures are being replaced by laparoscopic procedures. The demand for nurses and other health care professionals cannot keep pace with the increased need for health care required by the growing older population. Care will focus on the unique lifestyles and values of a diverse population. DIF: Comprehension REF: p. 376 2. Which factors would be considered in the first steps in developing an effective patient classification system? (select all that apply) a. Planned procedures b. Ethnic diversity of patients c. Clinical competency of staff d. Educational level of nurses e. Age of patients ANS: A, B, E The first step in developing a patient classification system is to understand the intensity of care needs, which requires identifying specific patient characteristics and care requirements. DIF: Comprehension REF: p. 368 3. A nurse responsible for staffing a medical-surgical unit must consider: (select all that apply) a. the patient census. b. physical layout of the unit. c. complexity of care required. d. educational level of all staff. e. task preferences of the nurses. ANS: A, B, C, D The primary considerations for staffing a specific nursing unit are the number of patients; the level of intensity of care required by those patients (commonly referred to as patient acuity); contextual issues, such as architecture, geography of the environment, and available technology; level of preparation and experience of the staff members providing the care; and the quality of the nurses’ work life. DIF: Application REF: pp. 367-368 Chapter 22: Quality Improvement and Patient Safety Cherry & Jacob: Contemporary Nursing: Issues, Trends, and Management, 7th Edition MULTIPLE CHOICE 1. A nurse is preparing to administer a medication by using the vastus lateralis site and is unfamiliar with the process. A step-by-step reference that shows how to complete the process is called a: a. deployment flowchart. b. top-down flowchart. c. Pareto chart. d. control plot. ANS: B A top-down flowchart shows the sequence of steps in a job or process such as medication administration. DIF: Comprehension REF: p. 379 |p. 388 2. A nonprofit organization that distributes to governmental agencies, the public, business, and health care professionals knowledge related to health care for the purpose of improving health is the: a. Institute for Safe Medication Practices. b. Institute of Medicine. c. National Committee for Quality Assurance. d. The Joint Commission. ANS: B The Institute of Medicine is a nonprofit organization whose mission is to advance and disseminate to the government, the corporate sector, the professions, and the public scientific information that will improve human health. DIF: Comprehension REF: pp. 379-380 3. A nurse is removing a saturated dressing from an abdominal incision and must cut the tape to remove the dressing. The nurse accidentally cuts the sutures holding the incision, and evisceration occurs. In quality improvement, this incident is best identified as a: a. root cause. b. sentinel event. c. variation in performance. d. causal factor. ANS: B A sentinel event is an unexpected occurrence that could result in serious physical or psychological injury to the patient, including the possibility of returning to surgery and a prolonged length of stay. DIF: Comprehension REF: p. 380 4. A nurse is assisting with the delivery of twins. The first infant is placed on the scale to be weighed. The physician requests an instrument stat. The nurse turns to hand the instrument to the physician, and the infant falls off the scale. When evaluating the incident, the nurse and her manager list contributory factors such as the need for two nurses when multiple births are known, and the location of the scale so far from the delivery field. These nurses are performing a(n): a. standardization of care. b. root cause analysis. c. process variation. d. analysis of a deployment flowchart. ANS: B A root cause analysis is a process by which factors that underlie variation in performance, including the occurrence or possible occurrence of a sentinel event, are identified. The purpose of root cause analysis is to identify improvements that can be implemented to prevent future occurrences. DIF: Application REF: p. 380 5. Each month data on admission assessments that are based on the following standard are entered: “All patients will be assessed by an RN within 2 hours of admission.” The target goal for this standard is 97% compliance. Data are displayed on a graph that shows number and time of admission assessments and compliance variation limits. This pictorial representation is: a. Pareto chart. b. control chart. c. deployment chart. d. top-down flowchart. ANS: B The control chart is a run chart that has a centerline and added statistical control limits that help to detect specific types of change needed to improve a process. DIF: Comprehension REF: p. 380 6. Regardless of the term used to describe high-quality health care, the focus of quality is: a. what the consumer needs and wants. b. economical care. c. having the greatest technologic advancement. d. services equally distributed among populations. ANS: A The customer determines quality on the basis of his or her unique perception of high-quality care. DIF: Knowledge REF: p. 382 7. A team of experienced nurses work together to develop algorithms that are converted into checklists to ensure standardization of commonly performed procedures. The focus of this team is primarily on which Institute of Medicine (IOM) competency? a. Safety b. Timely c. Equitable d. Patient-centered care ANS: A Standardization contributes to safety and improves individual performance of care providers. DIF: Application REF: p. 381 8. An organization’s emergency preparedness task force meets to discuss how it should react in case of a terrorist attack and develops a disaster evacuation plan that details how each department will assist individuals in reaching safety. This type of diagram is referred to as a: a. Pareto chart. b. control chart. c. top-down flowchart. d. deployment chart. ANS: D A deployment flowchart would show the detailed steps involved in the process and the people or departments that are to be involved at each step to assist individuals in reaching safety. DIF: Comprehension REF: p. 385 9. Patients with heart failure have extended lengths of stay and are often readmitted shortly after they have been discharged. To improve quality of care, a type of “road map” that included all elements of care for this disease and that standardized treatment by guiding daily care was implemented. This road map is referred to as a(n): a. benchmark. b. critical pathway. c. algorithm. d. case management. ANS: B A critical pathway determines the best order and timing of interventions provided by health care team members for a particular diagnosis. DIF: Knowledge REF: pp. 388-389 10. The staff on a nursing unit notes that patient satisfaction varies from month to month. They plot the degree of patient satisfaction each month for 1 year to determine when the periods of greatest dissatisfaction are occurring. The staff uses which type of graph? a. Time plot b. Pareto chart c. Flowchart d. Cause-and-effect diagram ANS: A A run plot, or time plot, graphs data in time order to identify any changes that occur over time. DIF: Comprehension REF: p. 388 11. A group of nurses is presenting the importance of high-quality care during a system-wide meeting of medical-surgical nurses. They point out a finding of the Quality Chasm that: a. being insured has little effect on a person’s longevity and the quality of care received. b. lobbyists for the drug companies are able to gain permission for the use of new drugs within 1 year of their discovery. c. although a greater number of lawsuits stem from medication errors, more people actually die from human immunodeficiency virus (HIV) and acquired immunodeficiency disease syndrome (AIDS). d. medication-related errors place a tremendous financial burden on the U.S. health care system. ANS: D Medication-related errors for hospitalized patients cost roughly $2 billion annually. DIF: Comprehension REF: pp. 381-382 12. According to the Quality Chasm report: a. health care providers should be proactive rather than reactive to patient needs. b. common needs rather than individual preferences should be the priority. c. medical information should be confined to the primary care provider. d. specialized providers or case managers should control health care decisions. ANS: A Quality is based on predicting patient needs rather than reacting to needs. DIF: Comprehension REF: pp. 381-382 13. During the night, a patient fell in the bathroom and sustained a hip injury. The patient was very upset because of being unable to attend a granddaughter’s wedding in 2 days. The team looked at the process and determined that the patient had been medicated with a narcotic, had urinary urgency so had not taken the time to put on shoes, failed to turn on the light because the door to the hall let in some light, and stumbled over a towel that had been placed to collect water leaks caused by construction that was in progress to replace damaged sinks. Which factor was a special cause variation? a. Failure to take time to put on shoes due to urgency b. Unsteady gait due to narcotic administration c. Poor lighting that led to decreased vision d. Improper construction that caused the leak and towel placement ANS: D A special cause variation is an uncommon variation that is unstable and unpredictable, is not under statistical control, and is related to a clearly identified single source, which in this scenario is the construction project. DIF: Application REF: p. 383 14. The number of IV site infections has more than doubled on a nursing unit. The staff determine common causes include the site is cleaned using inconsistent methods, dressing frequently becomes wet when patient showers, IV tubing is not changed every 48 hours per protocol, and inadequate hand washing of RN prior to insertion. A bar graph demonstrates the frequency in descending order, with 80% of infections being attributed to inadequate hand washing. The quality tool used is a: a. cause-and-effect diagram. b. run chart. c. Pareto chart. d. flowchart. ANS: C Pareto charts are bar graphs that show causes contributing to a problem in descending order so the leading cause is easily recognized. DIF: Comprehension REF: pp. 385-386 15. The surgical team arrives in the operating room and one member states, “Everyone stop. Let’s identify the patient and operative site. Now does anyone have any questions or concerns?” This process is known as: a. time-out. b. a critical pathway. c. special cause variation. d. lean methodology. ANS: A A time-out occurs in the operating room to ensure the entire surgical team identifies the patient, operative site, and possible concerns or questions about the procedure. DIF: Comprehension REF: p. 391, Box 22-2 16. Institute for Healthcare Improvement (IHI) proposed a process for quality improvement with steps known as “PDCA.” When explaining the steps to a group of nurses interested in improving the process of medication reconciliation for heart failure patients with high rates of recidivism, the instructor states: a. P stands for process. Following a top-down flowchart provides the steps for reviewing patient medications taken at home compared to those prescribed during hospitalization. b. D stand for deviation, which is an alteration in the expected drugs ordered. c. C is for check if the process for change worked. Was there an improvement in accurate reconciliation? And what was learned? A stands for algorithm, which includes all steps of the process. d. A stands for algorithm, which includes all steps of the process. ANS: C C stands for check if the change improved the process and what was learned. DIF: Analysis REF: p. 397, Figure 22-7 17. 17. A hospital is concerned that the number of medication errors has increased significantly in the past year. A project revealed four causes of medication errors. The above chart was used to help staff and administration know where to focus efforts to reduce errors. Which process improvement tool is used in this situation? a. Run chart b. Pareto chart c. Flowcharts d. Cause-and-effect diagrams ANS: B Pareto charts are used to prioritize areas to reduce medication errors. Eighty percent of all errors were caused by interruptions, so this should be the area of priority. DIF: Comprehension REF: pp. 385-386 18. A nurse is asked to “float” to a telemetry floor and is to place a patient on telemetry monitor. The nurse is unfamiliar with placement of EKG leads and would consult which type of chart to learn the correct placement? a. b. c. d. ANS: C The Pareto chart is used to prioritize interventions that caused the majority of the problems. DIF: Application REF: p. 387, Figure 22-3 19. Which of the following occurrences would be classified as a sentinel event? a. A postpartum patient who elects to breastfeed only twice daily develops mastitis. b. A newly diagnosed diabetic patient self-injects insulin in the abdominal area rather than the upper thigh as instructed by the patient educator. c. A nurse assisting with the delivery of twins places the “Twin 1” name tag on the second-born twin, causing the first-born twin to undergo surgery that was scheduled for the other twin. d. A nurse administers 3 units of regular insulin rather than 3 units of NPH insulin subcutaneously, resulting in a drop in the patient’s serum glucose from 160 to 100 mg. ANS: C Any procedure performed on a wrong person or organ constitutes a sentinel event. DIF: Application REF: p. 380 |p. 391 20. A patient is ordered a low-protein, low-calorie diet but the patient’s family brings fish, lentils, and unleavened bread for a meal to observe a cultural practice. The nurse works with the dietitian to adjust the next few meals to accommodate for this variance. This situation would represent: a. a sentinel event. b. an adverse event. c. patient-centered care. d. the communication technique of “call-out.” ANS: C The nurse and dietitian are respecting patient values, preferences, and expressed needs. DIF: Comprehension REF: pp. 381-382 21. Nurses, physicians, and social workers finalize the plan of care and coordinate discharge for a homeless person who will need wound care and follow up over the next 4 weeks. Each member contributes based on his or her area of expertise but also recognize other members’ strengths. Which of the QSEN competencies are being demonstrated? a. Quality improvement b. Evidence-based practice c. Teamwork and collaboration d. Patient-centered care ANS: C An interdisciplinary team is working to prevent hand-off errors on discharge. DIF: Application REF: p. 393 22. Which of the following statements concerning the Institute of Medicine (IOM) competencies is correct? a. Each competency is mutually exclusive. b. The competencies focus on individual efforts to reduce errors. c. Physicians lead the team to achieve each competency. d. The competencies address both individual and system approaches to transform care. ANS: D Errors and increased health care costs result from both the actions of health care workers and the nature of the system in which they deliver care. DIF: Comprehension REF: p. 381 23. Nurses working on an orthopedic unit use personal digital assistants (PDAs) to review medications prior to administration to reduce potential drug interactions. Software is also installed that provides video clips of common procedures performed by nurses. Nurses on this unit are best demonstrating which QSEN competencies? a. Patient-centered care b. Informatics c. Teamwork d. Quality improvement ANS: B Technology (PDA) is used to aid decision making and reduce errors. DIF: Comprehension REF: p. 393 MULTIPLE RESPONSE 1. A patient with complicated diabetes is scheduled for a below the knee amputation at 7 AM. The surgical team adheres to the 2012 National Patient Safety Goals by implementing which protocols? (select all that apply) a. The surgical team asks the patient to verify his or her name, type of surgery, and limb to be removed. b. Ask each member of the surgical team to provide a copy of licensure and, if applicable, certification to patient and family. c. The surgical team uses the chart number and name/hospital number to ensure they have the correct patient. d. Mark the procedure site with “X” and again ask the patient to verify correct site. e. After arrival in the operating room, perform a “time-out” for final identification of patient and operative site along with agreement of what procedure is scheduled. ANS: A, C, D, E The 2012 National Patient Safety Goal includes universal precautions to ensure patient safety and prevent sentinel events. Methods to identify patient and surgical procedure are required. DIF: Application REF: p. 391, Box 22-2 2. An interdisciplinary team is evaluating the hospital’s care of patients admitted with a myocardial infarction (heart attack) compared to national standards. The team analyzes the hospital’s clinical indicator, which would be: (select all that apply) a. aspirin order within 24 hours of discharge. b. patient teaching related to stopping smoking completed prior to discharge. c. beta blocker administered upon arrival. d. support of employer to modify stress in workplace. e. patient’s willingness to adhere to a strict cardiac diet after discharge. ANS: A, B, C Clinical indicators are measurable items that reflect the quality of care provided and demonstrate the degree to which desired clinical outcomes are accomplished. National benchmarks are established according to guidelines related to quality care for patients admitted with heart attack and include: aspirin within 24 hours of admission, angiotensin receptor blocker at discharge, stop smoking instruction given, and beta blocker administered upon arrival and discharge. These are all measurable. DIF: Application REF: p. 385 3. A nurse educator is explaining to licensed staff that health care is no longer safe and describes The Quality and Safety for Nursing (QSEN) recommended competencies for educating nursing professionals. These include: (select all that apply) a. advanced health assessment techniques. b. patient-centered care. c. prescriptive pharmacology content. d. quality improvement. e. safety. ANS: B, D, E Patient-centered care is a recommended competency, along with teamwork and collaboration, evidence-based practice, and informatics. Quality improvement is a recommended competency, along with patient-centered care, teamwork and collaboration, evidence-based practice, and informatics. Safety is a recommended competency, along with patient-centered care, teamwork and collaboration, evidence-based practice, and informatics. DIF: Comprehension REF: p. 393 COMPLETION 1. Quality is defined by the . ANS: patient Quality is based on the perspective of the consumer or, in this instance, the patient. DIF: Knowledge REF: pp. 382-383 2. While taking a shower, a patient pushes the emergency light. When the nurse arrives, the patient complains of feeling dizzy and unsteady. The nurse turns to reach for the patient’s walker and the patient falls, hitting the right side of the face resulting in loss of vision in the right eye. This scenario represents a event. ANS: sentinel A sentinel event is an occurrence that results in death or serious illness and requires immediate investigation. DIF: Comprehension REF: p. 380 Chapter 23: Health Policy and Politics: Get Involved! Cherry & Jacob: Contemporary Nursing: Issues, Trends, and Management, 7th Edition MULTIPLE CHOICE 1. A bill is pending in the state legislature that will mandate teaching health promotion related to smoking cessation. Which action by the nurse should have the greatest impact on passage of the bill? a. Explain the rationale for the bill to the school board. b. Contact the elected representative’s office by telephone to request support for the bill. c. Support a health advocate to run for election as state representative. d. Tell all students about the bill. ANS: B Telephone calls are an effective means of communicating your support for a bill, next to a face-to-face meeting; ask to speak to the staff person assigned to the bill or issue for which the call is being made. After introducing yourself, give a brief and simple message such as, “Please tell Senator/Representative [name] that I support [bill number].” DIF: Application REF: p. 407 2. Time is becoming crucial for the nurse to address issues related to sponsoring programs to include hearing aid costs as part of Medicare coverage. The nurse involved in grassroots political actions realizes that the least timely method of reaching elected officials would be via: a. letter. b. telephone. c. e-mail. d. facsimile (fax). ANS: A Sending letters is the least timely mode by which to communicate with elected officials; because of the anthrax decontamination process that is now in place, delivery of U.S. mail to Congress and to the White House may be delayed by as long as 3 months, making letters an inefficient means of communicating with policymakers. DIF: Comprehension REF: p. 407 3. A nurse would like to advocate for increased protective services and reporting mechanisms for elder abuse and attends the “meet the candidate” session at the town hall meeting. This is an important time for the nurse to: a. educate the public about the nurse’s political platform. b. be spontaneous and not deliver a rehearsed speech. c. address the person as “candidate” rather than using a first name that implies a working relationship. d. learn what the key issues are in the candidate’s platform. ANS: D Town hall gatherings with nurses allow the candidate to talk about his or her platform to a group of interested voters and afford nurses an opportunity to understand the candidate’s vision and to voice their opinions and concerns about health care issues. DIF: Application REF: p. 406 4. A nurse would like to obtain background information and learn the political platform of a candidate who is running for national office. The nurse should contact the: a. National Registry for Candidates. b. American Nurses Association Nurses Strategic Action Team (N-STAT). c. National League of Nurses. d. National Council of State Boards of Nursing. ANS: B Through legislative updates, N-STAT keeps members up-to-date about background information and the platforms of candidates who are running for national office; N-STAT also provides updates about key bills as they move through the legislative process. DIF: Comprehension REF: p. 408 5. Political action committees (PACs) are established for the purpose of: a. financially supporting candidates. b. persuading policymakers to support certain policies. c. lobbying legislators. d. recommending health care policy. ANS: B A PAC is an arm of an organization, association, or labor union that is formed to persuade a policymaker to support a certain policy or program or, more often, to ensure the election or reelection of policymakers who support the organization’s goals. DIF: Comprehension REF: p. 406 6. When the policy process is compared with the nursing process, identifying the issue is consistent with which step of the nursing process? a. Assessment b. Diagnosis c. Planning d. Implementation ANS: B The diagnosis step in the nursing process is consistent with identification of the targeted issue in the political process. As politically active nurses soon discover, effective involvement in policy development and political activities requires efforts similar to those used in the nursing process. The policy process and the nursing process are systematic approaches that use the nursing process for decision making. DIF: Application REF: p. 405 7. It is important for nurses to know the functions of the branches of the federal government. At a local meeting of the state nurses’ association, an officer reminds members that the branch that is capable of originating major policy initiatives is the branch. a. executive b. legislative c. judicial d. administrative ANS: B The legislative branch possesses the sole federal power to enact legislation; the legislative branch originates and promotes major policy initiatives and has the power to override a presidential veto. DIF: Comprehension REF: p. 403, Table 23-2 8. A concerned nursing student calls the office of an elected official to voice support for the bill to ban smoking in all public places. The secretary asks, “Are you a constituent?” What does the term constituent mean? a. A member of a professional organization who supports the organization’s political issue b. A citizen who is registered and is eligible to vote for a representative c. An elected official who proposes legislation to be considered as a potential law d. A member of the House of Representatives who is seeking support for a particular bill ANS: B A constituent is a citizen who has the opportunity to vote for candidates in elections for representation at local, state, and federal government levels. DIF: Knowledge REF: p. 398 9. A person who is covered by Medicaid moves from one state to another and asks the nurse at the health department why the benefits changed. The correct response by the nurse is: a. “Local government controls which benefits each county can provide to Medicaid recipients.” b. “The federal government does not fund any part of Medicaid, so each state must find ways to pay for benefits.” c. “Each state determines benefits on the basis of the present economy.” d. “Medicaid is funded by both state and federal governments, but benefits vary from state to state.” ANS: D Medicaid is a program that is funded through a combination of state and federal funds; thus benefits vary from state to state. DIF: Analysis REF: p. 400 10. Throughout the history of health care in our nation, one particular piece of federal legislation provided funds for hospital construction. As the number of hospitals rapidly increased, the need for nurses to staff hospitals also increased, and the shift from community-based nursing care to hospital-based nursing care began. This fundamental piece of legislation was the: a. Sheppard-Towner Act. b. Hill-Burton Act. c. Veterans Bill. d. Access to Health Care Bill. ANS: B The Hill-Burton Act, also known as the Hospital Survey and Construction Act, was enacted in 1950. This act provided funding that resulted in a boom in the construction of hospitals across the country. As the number of hospitals increased rapidly, so did the need for nurses to staff the hospitals. Thus the nurse’s role was shifted from community and public health settings to the acute care setting. DIF: Comprehension REF: p. 401 11. Members of a state’s constituent member association meet at a local restaurant to plan a Nurse Walk to support the local food bank. When entering the restaurant, the nurses note the environmental inspection report shows deficiencies because food handlers were not wearing hairnets and rodents were spotted, resulting in a score of 60 out of 100. This certificate of inspection is health policy enforced at which level of government? a. Local b. State c. Federal d. International ANS: B Health policy at the state level is responsible for ensuring food safety in restaurants. DIF: Comprehension REF: p. 400 12. In the near future, the largest segment of the population will be those over age 65 years. To advocate for this vulnerable population, a nurse was a strong supporter for the Medicare Modernization Act (Medicare Part D), which: a. provided vision care benefits to elderly persons who were diagnosed as legally blind. b. removed limits to access to mental health/substance abuse services which are the most commonly used service in this population. c. provided coverage for medications for Medicare enrollees that constituted a huge expense for the elderly. d. expanded the practice opportunities for advanced practice registered nurses (APRN) who provide the majority of care for this population. ANS: C The Medicare Modernization Act provided medication benefits to Medicare recipients and encouraged policymakers to have oversight to ensure drug effectiveness. DIF: Comprehension REF: p. 401, Table 23-1 13. If a nurse practicing in a mental health agency could only belong to one professional organization, what criteria could be used to select the organization with the biggest impact? a. The American Nurses Association has a strong voice representing the nation’s entire RN population. b. Specialty organizations such as the American Psychiatric Nurses Association lobby for higher education requirements for nurses. c. State nursing associations have limited power because they can lobby only state representatives. d. Professional organizations such as The American Association for the History of Nursing (AAHN) record the history of mental health nursing. ANS: A The American Nurses Association is the voice for all registered nurses. DIF: Application REF: p. 408 14. A group of nurses meet with a state representative to explain the importance of allowing registered nurses the right to sign death certificates for patients who were under their care. As constituents and nurses, they requested that the senator vote against the bill restricting this practice. These nurses are serving as: a. lobbyists. b. policymakers. c. officials with regulatory power. d. advocates for the Patient Safety and Quality Improvement Act. ANS: A Lobbyists hope to educate and convince policymakers to respond positively to a particular position on an issue or to follow a particular course of legislative or regulatory action. DIF: Comprehension REF: p. 398 MULTIPLE RESPONSE 1. A nurse is concerned about older persons living alone at home without telephone service to contact emergency services. The nurse is advised to take part in grassroots efforts to address the situation. The nurse would: (select all that apply) a. post signs to support the county’s political candidates whose platform support services for poor and underserved person such as older adults. b. visit with local politicians running for office and ask specific questions such as, “Would you support providing free emergency telephone service for older adults living alone?” c. limit voting to only presidential elections in which a candidate promises to initiate health care reform related to Medicare for older adults. d. join the American Nurses Association and constituent member association. e. contact representatives from surrounding states to garner support for free medical alert systems for older adults. ANS: A, B, D Working in local political campaigns is an effective grassroots effort. Visiting, e-mailing, calling, and faxing local politicians to support free emergency telephone service for older adults is a grassroots tactic. Joining ANA and state nursing associations supports these organizations’ lobbying efforts and is thus a way for individual nurses to influence the issues that nursing associations support. DIF: Application REF: pp. 405-408 2. A nurse is interested in learning more about health policy and how it might impact nursing practice. She is especially interested in The Patient Protection and Affordable Care Act (PPACA) and how it will travel through the three branches of the federal government. She researches the roles of the legislative branch of the federal government and learns that this branch: (select all that apply) a. determines which laws are constitutional, such as recently deciding the PPACA is constitutional because it is a form of taxation. b. has the exclusive power to determine where federal dollars will be spent, such as supporting the PPACA. c. consists of the Office of the President and 15 executive departments that supported the PPACA. d. has the power to veto laws found constitutional by the Supreme court and therefore could veto the PPACA. e. can override a Presidential veto, making it possible to amend the PPACA even without approval of the President. ANS: B, E The legislative branch possesses the sole power to enact legislation, tax citizens, and allocate federal spending. The legislature can override a presidential vote. DIF: Application REF: p. 403, Table 23-2 COMPLETION 1. A nurse who is interested in graduate school wants to learn about recently passed legislation regarding changes in Medicare reimbursement for care provided by clinical nurse specialists. The best source for information on changes in federal programs is the . ANS: Federal Register The Federal Register is the best source of information about proposed rules and regulations for newly enacted legislation and about changes to existing rules for federal programs (www.fr.cos.com). DIF: Knowledge REF: pp. 404-405 2. The greatest barrier to access to health care for a poor Hispanic person living in downtown Chicago is lack of health . ANS: insurance Lack of health insurance is the greatest barrier to access to health care and it has a tremendous impact on an individual’s health. Studies have consistently found that the uninsured receive less than adequate health care. DIF: Comprehension REF: p. 409 3. Another name for an eligible voter is a . ANS: constituent DIF: Comprehension REF: p. 398 Chapter 24: Making the Transition from Student to Professional Nurse Cherry & Jacob: Contemporary Nursing: Issues, Trends, and Management, 7th Edition MULTIPLE CHOICE 1. During orientation, a novice nurse sits and “virtually spends” the first few paychecks, envisioning the money going into a personal bank account. In the dream state, the nurse smiles and knows that the pain of nursing school was worth it. Which phase of reality shock is the nurse experiencing? a. Honeymoon b. Shock or rejection c. Recovery d. Resolution ANS: A During the honeymoon phase, the novice nurse has a positive image of nursing that coincides with the reason for becoming a nurse; this feeling is experienced immediately after the nurse begins work and often while still in orientation. DIF: Comprehension REF: pp. 416-417 2. A novice nurse is assigned a patient who has an order to draw blood for culture and sensitivity from a central line before antibiotic therapy is started. The novice reads and rereads the procedure manual. An hour later he stands at the bedside of the patient and stares at the central line, without knowing how to proceed. This phase of reality shock is termed: a. honeymoon. b. shock or rejection. c. recovery. d. resolution. ANS: B During the shock or rejection phase, there is inconsistency with what was learned in school and the work environment, and the novice nurse lacks many of the skills needed to be independent in this new role. DIF: Comprehension REF: pp. 416-417 3. According to Kramer, nurses in the shock phase should ask themselves: a. “How can I fit in with other staff?” b. “Why can’t I perform as everyone expects?” c. “What changes can I make to make me feel good about the choice to become a nurse?” d. “How can I get the rest of the staff to change?” ANS: C During the shock or rejection phase, novice nurses must ask themselves what they must do to become the type of nurse envisioned and to make a contribution. DIF: Application REF: pp. 416-417 4. The novice nurse arrives on the nursing unit, is introduced to the staff, is assigned a preceptor, and is asked to participate in the next staff meeting. This introduction into nursing is appropriately termed: a. biculturalism. b. socialization. c. transition. d. “going native.” ANS: B Socialization involves the acceptance and integration of the novice nurse into the profession of nursing, as well as the identification of the novice nurse with the profession of nursing. DIF: Comprehension REF: p. 414 5. Several novice nurses share lunch breaks and have comments such as, “I have insomnia from worrying about what I forgot to do for my patient,” and “I have no energy,” and “I can’t believe I can’t do my job correctly.” These novice nurses are each experiencing different symptoms of: a. burnout. b. low self-esteem. c. lack of confidence. d. resilience. ANS: A Symptoms of burnout include extreme fatigue, headaches, difficulty sleeping, mood swings, anxiety, poor work quality, depression, and anger. DIF: Comprehension REF: pp. 417-418 6. Which nurse is more prone to burnout? The nurse who: a. graduated last in the class. b. is industrious and conscientious. c. refuses to work an extra shift even though he or she needs the money. d. volunteers to serve on only one committee and keeps in touch with schoolmates. ANS: B Burnout is more common among nurses with type A personalities and those who are overachievers. DIF: Application REF: pp. 417-418 7. A novice nurse notices the medication nurse hurriedly gathering medications to be administered for the entire day, removing them from the packages, and placing them in a paper cup with the patient’s name and room number on the cup. When one of the patients points at a pill and asks, “What is this?” and “What does this do?” the medication nurse is uncertain, because it has long since been removed from the packaging. The novice nurse speaks to the nurse manager about her observations and describes her concerns. The novice nurse has assumed the role of: a. loner. b. “rutter.” c. change agent. d. “native.” ANS: C The novice nurse is working to improve the safety and patient environment and is a patient advocate. DIF: Comprehension REF: p. 418 8. The novice nurse calls a supervisor and requests that a “float nurse” be assigned to help with all “assessments of new admissions and postoperative patients.” The supervisor asks, “How many admissions and surgery patients have you received?” The novice nurse becomes flustered because she has not checked the census but simply bases the need on feeling overwhelmed. This situation best indicates a lack of skills. a. organizational b. communication c. interpersonal d. clinical ANS: A The novice nurse lacks proficiency, which may be exaggerated by feelings of being overwhelmed by the new environment, causing him or her to not get the facts before asking for help. DIF: Comprehension REF: pp. 422-423 9. When the novice nurse asks, “What will happen if this task is not completed,” which skill is being demonstrated? a. Priority setting b. Delegation c. Organization d. Clinical skills ANS: A The novice nurse is considering the legal and safety ramifications if the task is not completed. DIF: Comprehension REF: pp. 423-424 10. In distinguishing between evaluation methods used in school versus those used in the work environment, it is noted that the work environment evaluation includes: a. determining whether steps are logical. b. formulating increments in correct order. c. efficiently organizing stages of the procedure. d. appraising outcomes according to policy. ANS: D Outcomes are based on meeting existing criteria rather than on knowing how the outcome was achieved. DIF: Application REF: p. 420 11. When comparing mentoring and role modeling, the role of mentoring is represented by which of the following situations? a. The experienced nurse is unaware that the novice nurse is observing him or her gathering supplies needed to start intravenous antibiotics. b. The novice nurse mimics conversations with his or her own patients that were unobtrusively overheard between the experienced nurse and his or her patients. c. The novice nurse receives feedback from the experienced nurse related to the use of a new occlusive dressing product and is told, “I couldn’t have done it better myself.” d. An experienced nurse is nominated for outstanding employee of the month by the novice nurse. ANS: C The experienced nurse knowingly enters into a relationship to advocate for and to enhance the self-esteem of the novice nurse. DIF: Analysis REF: p. 425 12. A certified oncology nurse notices that a novice nurse is unsure of decision making and lacks technical skills. The novice nurse gains confidence by sharing and learning with the experienced nurse. This relationship continues and builds, allowing the novice nurse to become more confident. This relationship is known as: a. mentoring. b. role modeling. c. a preceptorship. d. socialization. ANS: A The experienced and novice nurses have entered into a mutually agreed upon interactive relationship to ease the transition of the novice nurse into the profession. DIF: Comprehension REF: p. 425 13. A novice nurse is unsure how to correctly administer an injection using the Z-track method. What is the best approach for learning this procedure? a. Read the procedure manual and follow the steps exactly. b. Make an appointment at the skills laboratory of the former nursing school to practice. c. Ask to observe the skill as it is being performed; then perform it under direct supervision. d. Try to remember how the task was previously performed during a simulation. ANS: C The novice nurse can become familiar with the technique, then can perform the skill under the supervision of an experienced nurse, ensuring competency. DIF: Application REF: p. 429 14. A student nurse wants to meet other nursing students from different countries and to learn of employment possibilities. The student nurse should: a. participate in the clinical facility’s employee satisfaction task force. b. join the Student Nurses Association. c. take part in self-mentoring. d. postpone taking the licensure examination so he or she can take a tour of foreign countries. ANS: B Students who participate in preprofessional organizations such as the Student Nurses Association have an opportunity to meet students across the nation and around the world and can network with leaders to gain knowledge about employment possibilities. DIF: Comprehension REF: pp. 426-427 15. An experienced nurse working in oncology is shocked to realize that he feels little empathy when a patient explains, “I developed cancer from having to work in the dry cleaning industry since I was only 8 years old to help support my younger siblings during the depression. Now they have all moved away and refuse to help me financially, and it takes everything I have to pay for my chemotherapy medicine.” The nurse is experiencing: a. burnout. b. compassion fatigue. c. reality shock. d. horizontal hostility. ANS: B The nurse is experiencing a gradual decline in compassion over time as a result of being exposed to events that have distressed his or her patients, such as working at a young age to support a family while inadvertently being exposed to carcinogens. DIF: Comprehension REF: pp. 417-418 16. A recruiter is explaining benefits to a group of nursing externs who are highly sought for employment. The recruiter states, “We are the only hospital in town that offers a residency program.” The recruiter further explains that a residency program: a. provides housing for the graduate nurse to decrease expenses until income is established. b. partners a medical resident with a nurse resident to learn interprofessional care. c. allows new graduates to work on a higher degree in nursing while being paid full-time. d. offers extended time for both theory and clinical activities that promote problem solving and clinical decision making. ANS: D Residency programs offer a longer precepted orientation period (often 12 months) to bridge the gap between the classroom and practice. DIF: Comprehension REF: pp. 425-426 17. A novice nurse is stressed due to always being behind with her assignments. She is overheard saying, “No one here worries about checking nasogastric tube placement before they give medications and hang feedings. Skipping that step would save me an extra 30 minutes to be used for charting. If they can do it, so can I.” This nurse is experiencing which stage of reality shock? a. loner. b. “rutter.” c. change agent. d. “native.” ANS: D The novice nurse is mimicking the actions of the experienced nurse taking shortcuts that were not taught in school and not following policy and procedure. DIF: Comprehension REF: p. 417 18. A novice nurse is placed in charge just after orientation ends. Which statement by the novice nurse would ensure a smooth delegation experience when delegating to an experienced staff member? a. “You have the patients in rooms 1 to 7. These patients require little skill so you will be fine.” b. “Your assignments are posted in writing by the nurses’ station. Be certain all your tasks are completed so the next shift is not behind.” c. “You have patients in rooms 5 through 10 and I will administer all IV medications. I will let you know if I have to adjust. If you see you need help let me know.” d. “Everyone is capable of providing the best care, so please sign up for the patients you would like to be assigned to.” ANS: C The nurse is clearly stating the assignments, allowing for a need to adjust both to needs of the unit and the delegate’s needs. DIF: Application REF: pp. 422-423 19. A novice nurse is in the “rejection phase” of stress and wonders if she made the right decision when deciding nursing was her chosen career. Which strategy would help the novice nurse cope with transition into practice and reduce stress? a. When asked by the charge nurse to accept the new admission even though she had no discharges, the novice nurse stops and considers if this request is acceptable and safe, then accepts the assignment based on two patients having a low acuity level. b. Refusing to eat anywhere but the hospital cafeteria to ensure a balanced diet and time to socialize with other staff and visitors. c. Avoid wasting time by listening to coworkers’ problems or ideas on how to staff the unit during the upcoming holiday since the novice nurse’s time is valuable and better spent getting her own work completed. d. Make an effort to “win over” hostile or angry coworkers who find fault with the novice nurse’s work performance. ANS: A To reduce stress and cope with the rejection phase of transition, the novice nurse should think before answering—take a few minutes before answering and deciding what is best course of action. DIF: Analysis REF: pp. 416-417 MULTIPLE RESPONSE 1. In orientation, a new graduate is surprised to learn of workplace violence in health care agencies and asks, “What kind of violence is common in hospitals?” The educator describes workplace violence as: (select all that apply) a. care of patients admitted who are victims of domestic violence. b. experienced nurses withholding pertinent information from coworkers to portray them as incompetent. c. coworkers participating in an employee assistance program (EAP) to learn more about anger management for adult children living at home. d. care of persons being held for treatment prior to be incarcerated for committing homicide. e. statements such as “The new nurse has book sense but can’t perform an admission physical without the help of everyone on the unit.” ANS: B, E Withholding information from novices or coworkers to purposely cause them to appear inadequate or unskilled is considered workplace violence because of the anxiety and stress it produces. This should not be tolerated and should be reported. Such statements, which belittle employees, lead to job dissatisfaction, lack of teamwork, and burnout and are considered lateral or workplace violence. This should not be tolerated and should be reported. DIF: Application REF: p. 424 2. Faculty comes to class to present information about joining the Student Nurses Association (SNA). A student is heard saying, “Why join this organization? It is just a bunch of students like us. I’ll wait and join the real nursing organization after graduation.” This student is not aware of which benefits of SNA? (select all that apply) a. Members are eligible to sit for the licensure examination at a reduced rate. b. Leadership skills can be refined by working with other schools of nursing. c. The National SNA provides discounted rates for study abroad opportunities. d. A global view of nursing can be obtained through networking internationally. e. Lobbying efforts are a major benefit of belonging to SNA. ANS: B, D Leadership opportunities are available as officers and through conferences. SNA members work with other nursing students across the nation and internationally. DIF: Comprehension REF: pp. 426-427 3. A novice nurse notices a patient is pacing the floor and twisting his hands. When the nurse enters the room, the patient stares at her and mumbles, “Can’t anyone understand what I want?” while smiling and reaching out to shake the nurse’s hand. The nurse recognizes a risk for violence based on which clinical findings? (select all that apply) a. Glaring at the nurse b. Nervousness, twisting of hands c. Murmuring when talking d. Diagnosis of a terminal illness e. Isolation, desire to be alone ANS: A, B, C Nurses can recognize a risk for workplace violence by using the acronym STAMPEDAR (staring, tone of voice, anxiety, mumbling, pacing, emotions, disease process, assertive/nonassertive behavior, and resources). DIF: Application REF: pp. 428-429 COMPLETION 1. It is important for novice nurses to know that “self-esteem = self-confidence + self-_ .” ANS: respect DIF: Knowledge REF: pp. 426-427 Chapter 25: Managing Time: The Path to High Self-Performance Cherry & Jacob: Contemporary Nursing: Issues, Trends, and Management, 7th Edition MULTIPLE CHOICE 1. In deciding whether to say no to a request that involves a time commitment, the professional nurse must consider: a. the cost-benefit ratio. b. the time commitment of coworkers. c. ways to buffer saying no. d. personal preference. ANS: A The first step in learning the art of saying no is determining when to say it. The cost/benefit ratio of each opportunity must be evaluated in relation to the overall goal. If the activity will provide an overall benefit, obviously it must be given careful consideration. If it will not result in significant benefit, decline gracefully but emphatically. DIF: Application REF: p. 446 2. Positive time management skills include: a. maintaining an open-door policy. b. retaining all paperwork. c. returning all telephone calls immediately. d. scheduling daily activities. ANS: D Planning is the most important step in time management. Planning allows people to better use their time and can lead to closure in relation to those goals that will produce the greatest internal satisfaction. DIF: Comprehension REF: pp. 441-442 3. Time can be maximized to produce the best outcomes by: a. making the first hour of each workday productive. b. completing trivial tasks before performing important tasks. c. eliminating all recreational activities. d. omitting breaks until the entire task has been completed. ANS: A Making sure that the first hour of every workday is productive sets the tone for achieving goals. DIF: Comprehension REF: p. 447, Box 25-2 4. To conduct a productive meeting, the nurse should: a. provide each person all the time needed to discuss desired topics. b. cover all emotional topics first. c. create an agenda with specific times allotted for each agenda item. d. wait for latecomers and ask for their excuse for being late. ANS: C You need to preplan for the meeting so it can be conducted in a timely manner. One way to have a productive meeting is to prepare a meeting agenda with time allotted for each item and the name of the person responsible for reporting on each item. Send out the agenda prior to the meeting so attendees can come prepared. DIF: Application REF: p. 439, Box 25-1 5. When choosing to delegate, the nurse should delegate the task to the most qualified person or to the person he or she wishes to: a. develop. b. promote. c. punish. d. reward. ANS: A Delegating is giving other people tasks to be accomplished. The benefits of delegation involve (1) assisting in developing the initiative, skills, knowledge, and competence of others; (2) maintaining the level of responsibility and decision making of others; (3) freeing time for more important tasks; (4) extending results that can be accomplished from what one person can do alone to what he or she can manage through others; and (5) ensuring that completing the task is cost effective. DIF: Comprehension REF: p. 445 6. When using the ABC system of managing time, those items coded A include: a. calling the pharmacy to see whether a drug insert is available for a patient. b. checking to see why a ventilator is alarming. c. organizing the medication cart. d. writing memos to remind everyone to contribute to the boss’s birthday gift. ANS: B A items should stand out from other items because of their worth and high level of importance. A items are most urgent and may require more energy and time, but they should be completed before any of the B or C items are performed. DIF: Application REF: p. 441 7. When deciding whether and when a task should be completed, a nurse must: a. complete all tasks as they are thought of to prevent having to take time to consider which is most important. b. procrastinate and hope that someone will volunteer to do it. c. ask, “What will happen if I don’t complete the task now?” d. view large projects holistically and not as many small pieces. ANS: C Stopping to evaluate what is going on is important because you may have to readjust your plan and reprioritize in order to reach your goal. DIF: Application REF: p. 444 8. When planning, a nurse should: a. delay planning until the “first task of the morning” has been completed. b. recognize that rewarding oneself has a negative consequence. c. rotate between several tasks to stimulate creativity. d. remember that most tasks take longer than anticipated to complete. ANS: D One must consider many factors when planning a project. Some of the following factors influence how priorities should be established: (1) urgency of a situation, (2) demands of others, (3) closeness of deadlines, (4) existing time frame, (5) degree of familiarity with the task, (6) ease of task completion, (7) amount of enjoyment involved, (8) consequences involved, (9) size of the task, and (10) congruence with personal goals. When the use of time is considered, not all of these factors carry the same weight, and adjustments will be needed. DIF: Comprehension REF: p. 440 9. A novice nurse is unsure of his ability to insert a nasogastric tube for one of the assigned patients who is vomiting coffee ground emesis. The novice nurse waits, hoping that someone with more experience will volunteer to do the job, or he just waits until the end of the shift. This nurse is practicing: a. energy management. b. priority setting. c. procrastination. d. introspection. ANS: C Procrastination is evident when a person is faced with an unpleasant task, a difficult task, or a difficult decision. Usually procrastination is easily recognizable because it involves completing low-priority tasks rather than high-priority ones, and it always welcomes interruptions. Procrastination is the art of “never doing today what can be put off until tomorrow.” The result consists of less productivity, less internal satisfaction, and increased stress. DIF: Comprehension REF: pp. 443-444 10. A student nurse assigned to work with the charge nurse is given the opportunity to help revise the nursing assessment form. She receives several compliments from management and her nursing instructor for her creative suggestions. The student nurse enjoys the project and attention she is receiving and begins to prolong the conclusion of the project. Although she constantly adds new information, she filters this out slowly to others. The student nurse is subject to the time management obstacle of: a. need for perfection. b. fear of losing creativity. c. unclear goals. d. fear of completion. ANS: D Fear of completion is a time management obstacle that may occur if you are afraid of completing a project that is creative and fun. To overcome this obstacle, take the time to understand why you are not completing the task or major project that has been with you for some time. DIF: Comprehension REF: p. 437 11. An inexperienced nurse has heard of other novice nurses who take shortcuts in providing patient care. This nurse feels that this is unacceptable and that all tasks must be performed faultlessly, which leads to her inability to complete all assigned tasks. This nurse would benefit from the seminar “Obstacles to Time Management: How to Deal with: a. Creativity.” b. Perfectionism.” c. Failure.” d. Downtime.” ANS: B If you are a perfectionist and feel that everything should be completed at the same level of excellence, you are not keeping things in perspective. If you demand extremely high standards for every single task you undertake, you simply will not get everything done. DIF: Comprehension REF: p. 437 12. An experienced nurse volunteers to serve on a task force intended to improve the quality of care because she possesses excellent patient care skills and has selected a career goal of working in quality management. However, the nurse is unable to concentrate on any one task or issue and is unable to view the health care milieu sensibly. Which source of energy is missing in this nurse? a. Physical b. Mental c. Spiritual d. Emotional ANS: B Mental energy is the ability to maintain sustained concentration on a task, to move flexibly between broad and narrow issues, and to be internally and externally focused, as needed by the situation. It includes mental preparation, visualization, positive self-talk, effective time management, and creativity. DIF: Comprehension REF: pp. 438-439 13. After keeping a log of activities designed to improve time management, the nurse divides the distractions into internal and external sources. The nurse would classify which distraction as internal? a. Responding to recurring crises at work or in one’s personal life b. Unsuccessful attempts to communicate with the unit manager c. Talking with potential faculty candidates d. Being given unclear job responsibilities ANS: A An internal distraction is one that can be controlled only by the person affected; it is important for each of us to recognize and understand the distracters that inhibit our ability to complete tasks and to meet our objectives and goals. DIF: Application REF: p. 439, Box 25-1 14. A nurse realizes that much time is wasted during shift report when coworkers discuss personal items such as recent movies or department store sales. Which statement would help with time management during this critical interaction? a. “Let’s talk only about one movie you recently watched.” b. “I like to know about department store sales, but we need to get through this report, so let’s talk about sales as we walk between patient rooms.” c. “I know your shift has been busy. What went wrong?” d. “I have the list of patients; let’s start with revisions to the plan of care and scheduled activities for the next shift.” ANS: D The statement “I have the list of patients; let’s start with revisions to the plan of care and scheduled activities for the next shift” focuses the conversation during report, keeping personal conversations to a minimum. DIF: Application REF: p. 439, Box 25-1 15. A nurse takes a day to travel to a state park where she can sit by the river alone to reflect over recent events and put things into perspective. She recalls the time she was complimented for her ability to deal with a difficult patient and another time when she was criticized for lack of teamwork. This nurse is improving which source of energy? a. Emotional b. Spiritual c. Physical d. Mental ANS: B Spiritual energy can be increased by taking time to reflect over events in one’s life and allowing time to understand feelings associated with events/happenings. A quiet environment is essential for reflection. DIF: Comprehension REF: pp. 438-439 16. As part of a quality assurance project, a nurse is to round on every patient daily for 1 month to ensure the intravenous tubing is labeled with date hung and nurse’s initials. The nurse is hoping to be promoted to a full-time position within the quality department, and her success on this project is very important to her receiving the position. The nurse retains all other assignments, and she is often interrupted to help other staff or answer call lights. The nurse discusses her concerns with the manager, who frees her for 1 hour each day to make rounds. During this 1 hour each day the nurse will organize herself and environment by practicing the: a. art of “no detourism.” b. reducing stacked-desk syndrome. c. art of “wastebasketry.” d. managing e-mail and memo mania. ANS: A The art of “no detourism” focuses on one task until completed. DIF: Comprehension REF: pp. 442-443 MULTIPLE RESPONSE 1. A nurse is having difficulty managing assignments at work, which results in a feeling of “failure” and tasks that are not completed or that are not completed satisfactorily. The mentor suggests some tips for time management. These include: (select all that apply) a. focusing on activities to be completed, rather than on objectives. b. planning for tomorrow today. c. making certain that the last hours are the most productive in tying up loose ends. d. maintaining a log of how the nurse spends time (no need to worry about using complete sentences). e. picking five major objectives for the day and not stopping until they are achieved. ANS: B, D Planning for the future is an effective time management strategy. Maintaining a log of how you spend your time is an effective time management strategy. DIF: Analysis REF: p. 447, Box 25-3 2. A nurse is having difficulty keeping up with the six assigned patients and serving on the ethics committee. In order to take charge of both personal and work life, the nurse focuses on improving physical energy by: (select all that apply) a. telling herself, “I provide safe quality care and will provide this level of care to all my patients.” b. taking a deep breath and remaining calm to develop patience. c. keeping a bottle of water available to consume a minimum of 24 ounces each shift. d. going into the nurse’s break room every 90 minutes to eat a healthy snack and prioritize remaining care. e. enrolling in the hospital’s fitness program, Nurses Need Nurturing. ANS: C, D, E Drinking plenty of water promotes physical health. Taking a break every 90 minutes, especially during long shifts such as 12-hour shifts, provides a much needed time to relax and reorganize. Proper and frequent exercise promotes physical health. DIF: Application REF: p. 438 3. A nurse hopes to improve time management skills using the ABC prioritization approach. Which tasks would be prioritized as “B”? (select all that apply) a. Turn in time sheet due in 3 days. b. Review dress code policy to give feedback before appointment in the morning. c. Perform blood glucose test on a patient admitted with Kussmaul respirations and change in level of consciousness. d. Complete patient teaching prior to discharge in 2 hours. e. Review procedure for inserting a PIC line to assist with procedure later this morning. ANS: B, D, E Task that falls within Priority B includes the medium-value items, such as tasks that are not urgent but most likely will not “fall” off the list. Since the dress code must be reviewed and feedback constructed for a morning appointment, this would be level B, it is not urgent nor will it likely fall off the list. Completing patient teaching for a patient to be discharged in 2 hours must be completed but does not take priority over “A” tasks. Reviewing a procedure to be performed later in the day is Level “B.” DIF: Application REF: pp. 440-441 4. A nurse is hoping for a promotion and seeks the help of a coach to improve time management skills. The first task assigned by the coach is for the nurse to list all external distractors that prevent organization and completion of tasks. The nurse would include: a. waiting for oncoming shift to start walking rounds. b. failure to delegate tasks to the licensed practical nurse and unlicensed assistive personnel and instead, completing task herself. c. talking with a patient’s family member who is also a personal friend. d. spending time thinking about the vacation to the beach. e. using the Internet to read about a new drug that will be administered; then taking time to look for news about the profession, new legislation about the Patient Protection and Affordable Care Act, and new uniforms for the season. ANS: A, C a. Waiting, such as for meetings or oncoming shift, is an external distraction. c. Socializing with visitors is an external distraction. DIF: Application REF: p. 439, Box 25-1 COMPLETION 1. The most important step in time management is . ANS: planning It is important to plan before beginning any task, project, or day’s activities. Planning involves (1) setting goals and establishing priorities, (2) scheduling activities, and (3) making to-do lists. DIF: Comprehension REF: p. 440 Chapter 26: Contemporary Nursing Roles and Career Opportunities Cherry & Jacob: Contemporary Nursing: Issues, Trends, and Management, 7th Edition MULTIPLE CHOICE 1. The primary focus of the care provider role for the nurse involves: a. using the nursing process to provide guidance to the patient in recovery. b. encouraging consumer self-responsibility and emphasizing health promotion/maintenance. c. focusing on increasing satisfaction among professional nurses. d. advancing nursing practice through research. ANS: A The primary focus of the care provider’s role is to use the nursing process in caring for patients. DIF: Comprehension REF: p. 451 2. A nurse researcher is attempting to describe the demographics of today’s RNs and reports that: a. less than half of all RNs work in hospital settings. b. the average age for an RN is 50 years. c. approximately 25% of RNs are male. d. most RN are practicing with a bachelor’s degree. ANS: D A substantial number (55%) of nurses practice with a BSN DIF: Comprehension REF: pp. 454-455 3. A hospital standard states, “All intravenous (IV) sites must be rotated every 72 hours.” The nurse who assesses and records variations from this standard is fulfilling which role of nursing? a. Infection control b. Quality management c. Coordinator position d. Counselor ANS: B The basic premise is to ensure that outcomes in client care services are consistent with established standards. DIF: Application REF: p. 456 4. The nurse who organizes screenings for employees, assesses the work environment for hazards, and teaches health-promoting activities is employed in the field of: a. infection control. b. occupational health. c. informatics. d. transportation specialization. ANS: B The occupational health nurse organizes screenings for employees, assesses the work environment for hazards, and teaches health-promoting activities. DIF: Comprehension REF: pp. 457-458 5. The RN who identifies the best resources at the lowest cost to achieve optimal health outcomes for the client is fulfilling the role of: a. informatics specialist. b. educator. c. case manager. d. quality manager. ANS: C The role of the case manager includes coordination of resources to achieve the best outcomes based on quality, access, and cost. DIF: Comprehension REF: p. 459 6. A patient is admitted for a hip replacement complicated by uncontrolled diabetes and early dementia. The nurse coordinates referrals to an endocrinologist to establish an insulin protocol and organizes out-patient physical therapy in an Alzheimer’s assisted living facility upon discharge. The role of this nurse is the: a. informatics specialist. b. educator. c. case manager. d. quality manager. ANS: C The role of the case manager includes coordination of resources to achieve the best outcomes based on quality, access, and cost. DIF: Comprehension REF: p. 459 7. An RN testifies at a trial where domestic violence is being investigated. She had previously assisted with specimen collection and had assessed the victim. The nurse is involved in nursing. a. triage b. forensic c. flight d. entrepreneurial ANS: B The forensic nurse provides direct services to clients and consults with and serves as an expert witness for medical and law enforcement. DIF: Comprehension REF: pp. 459-460 8. The forensic nurse would most likely be involved in: a. performing an assessment of rape victim and collecting specimens needed for court. b. assigning a patient with chest pain to be seen first, followed by a patient with an insect bite of unknown origin with red streaks but no respiratory distress. c. assessing a patient at home by asking the patient to place a receiver over the pacemaker to evaluate capture of pacing stimuli and transmitting the results to another state where nurse located. d. performing pediatric life support at the scene of an accident to stabilize then transport via helicopter to the nearest trauma center. ANS: A Forensic nursing involves providing direct services to clients involved in abuse or victims of crimes and consults with medical and law enforcement. DIF: Application REF: pp. 459-460 9. A nurse in an acute care facility helps patients understand how to prevent diabetic neuropathies. This nurse is functioning in the role of: a. change agent. b. educator. c. manager. d. coordinator. ANS: B A nurse functioning in the role of educator helps patients understand the complications of their disease process. DIF: Comprehension REF: p. 451 10. A nurse notices that care is often withheld until an interpreter arrives, if the nurse is not multilingual. The nurse discusses this with the management team and suggests a three-part series be presented to the nursing staff to teach some commonly used medical terms in Spanish. This nurse is involved in the role of: a. change agent. b. educator. c. manager. d. coordinator. ANS: A The role of the change agent consists of assessing health care delivery issues and creating solutions to improve health care delivery. DIF: Comprehension REF: p. 451 11. A nurse tallies the patient satisfaction surveys and approves the use of an agency nurse because of the high census. This nurse is involved in which nursing role? a. Change agent b. Educator c. Manager d. Coordinator ANS: C The manager is the one who assesses the need for staffing and must justify this in a substantial way. Through this approach, the manager indirectly provides care for patients and families. DIF: Comprehension REF: p. 451 12. In differentiating between interprofessional and multidisciplinary: a. multidisciplinary refers to the care of many diverse patients by one discipline. b. interprofessional involves members of many disciplines working together to provide patient care to a distinct population. c. interprofessional refers to members of the same discipline working together to provide patient care to a distinct population. d. interprofessional is the use of multiple interventions by nurses with varying levels of education. ANS: B Interprofessional, as related to member roles, involves members of many disciplines working together to provide patient care to a distinct population. DIF: Knowledge REF: p. 450 13. A person from an underrepresented background in nursing is interested in entering nursing school and researches the ethnicity, age, and gender make-up of the nursing profession and finds: a. nursing is a career field that is primarily made up of women younger than 30 years. b. the number of male RNs has steadily increased and is equal to the number of female RNs. c. the majority of RNs are white or non-Hispanic. d. more than half of all RNs are ages 55 through 65. ANS: C The majority of RNs are white or non-Hispanic (75%). DIF: Knowledge REF: p. 454 14. A nurse who is participating in a health fair asks, “Other than nursing, what are some opportunities for careers in the health care professions, and what education is required?” The nurse informs participants that a baccalaureate degree is required as the minimum standard for the role of: a. speech therapist. b. occupational therapist. c. nurse-midwife. d. nurse anesthetist. ANS: B The baccalaureate degree is the minimum requirement for an occupational therapist. DIF: Knowledge REF: pp. 452-453, Box 26-2 15. A nurse is interested in practicing in a faith-based community; when researching the role of the parish nurse, the nurse finds that: a. certification is available at a basic level after a qualifying examination is completed and a set number of practice hours have been worked, as defined by the state’s nurse practice act. b. this advanced practice role focuses on health promotion and disease prevention. c. this role has no set standards or scope of practice but instead follows the doctrine of the denomination being served. d. this role is recognized by the American Nurses Association (ANA) as a specialty with a defined set of practice guidelines. ANS: D The role of parish nurse has become a recognized specialty in a growing professional practice. In 1998, the ANA established the scope and standards of this professional practice. DIF: Comprehension REF: p. 460 16. A nurse is responsible for determining the incidence rates of hospital-acquired diarrhea on a pediatric ward after receiving reports that five children admitted with uncontrolled asthma had developed diarrhea while hospitalized. The nurse is working in the role of: a. quality management. b. infection control. c. occupational health. d. forensics. ANS: B The infection control nurse is concerned with incidence of infections within the hospital. DIF: Comprehension REF: p. 456 17. A nurse working in a factory performs routine hearing tests for employees. The factory uses 90 dB as the safe limit for an 8-hour day’s exposure to machine noise rather than 85 dB as recommended by the Occupational Safety and Health Administration. The nurse is functioning in the role of: a. quality management. b. informatics nurse specialist. c. occupational health nurse. d. case manager. ANS: C The nurse in the role of occupational health focuses on health promotion and disease control of employees. DIF: Comprehension REF: pp. 457-458 18. People are living longer and with older age comes a higher risk of terminal illness. Many individuals have chosen to maximize their quality of life but stop aggressive life-sustaining treatments. Nurses who work with terminally ill patients and their families to provide end-of-life care, pain management, and family support are practicing in which role? a. Hospice b. Infection control c. Coordinator d. Case manager ANS: A Hospice nurses specialize in holistic care for the patient and family and end-of-life care to maximize quality rather than quantity of years of life. DIF: Comprehension REF: pp. 456-457 19. 19. A nurse explaining the above graph indicates the frequency of falls in 2011 to detect and possibly explain causes of variation. The peaks in June were attributed to regular staff vacations during which agency nurses were used. The nurse is employed in which role? a. Case Manager b. Quality Manager c. Forensic Nurse d. Occupational Health Nurse ANS: B The role of the quality manager is to improve quality of care and reduce errors. Knowledge of quality management tools is essential to the role. The run chart shown above is one such quality management tool. DIF: Application REF: pp. 458-459 20. A new graduate desires a position in labor and delivery; however, no position is available. What is the best advice for this nurse? a. Continue to study to keep skills current and wait for desired position. b. Return to college until a position in labor or delivery opens. c. Accept another nursing position and transfer into labor and delivery when a position is available. d. Practice obstetrical skills in a simulation lab then present evidence of competency to potential employer. ANS: C Entry level positions in labor and delivery may be difficulty to find. Accepting a position in another area of the hospital will allow the novice nurse an opportunity to improve her skills and develop clinical decision making as she waits for an open position in labor and delivery. DIF: Application REF: p. 455 21. An RN has critical care experience and is proficient in advanced cardiac life support and often helps to lead “codes” and is an expert clinician with superior health assessment skills. Enjoying a challenge of a fast-paced environment with autonomy, this nurse would best be suited for which nursing role? a. Flight nurse b. Home health nurse c. Hospice nurse d. Occupational health nurse ANS: A An experienced nurse with critical care experience and a desire to practice in a complex dynamic health care environment requiring clinical decision making would thrive as a flight nurse. DIF: Comprehension REF: p. 459 22. The role of the Clinical Nurse Leader (CNL) would include: a. diagnosing a patient with pneumonia and prescribing antibiotics. b. teaching members of a faith community how to avoid skin cancer and referring those with suspicious lesions. c. participating in the team who is investigating possible abuse of elderly persons by members of a gang known to sell illegal drugs. d. coordinating care of a patient admitted 4 days ago with stage IV heart failure and chronic bronchitis by revising care plan to include ambulation 50 feet with assistance. ANS: D The CNL is not an advanced practice nurse but is an advanced generalist who provides care and coordination of care for patients with complex needs and revises care plan to meet desired outcomes. DIF: Application REF: pp. 460-461 MULTIPLE RESPONSE 1. A nurse wants to become an advanced practice nurse and investigates the requirements for a: (select all that apply) a. nurse practitioner. b. nurse executive. c. certified nurse-midwife. d. certified registered nurse anesthetist. ANS: A, C, D A nurse practitioner is considered an advanced practice nurse. A certified nurse-midwife is considered an advanced practice nurse. A certified registered nurse anesthetist is considered an advanced practice nurse. DIF: Knowledge REF: pp. 461-462 2. Advanced practice nurses are prepared minimally at the master’s degree level with prescriptive privileges; these professionals include: (select all that apply) a. clinical nurse leader. b. nurse practitioner. c. nursing administrator. d. certified nurse-midwife. e. clinical nurse specialist. ANS: B, D, E A nurse practitioner requires preparation at the master’s degree level. A certified nurse-midwife requires preparation at the master’s degree level. A clinical nurse specialist requires preparation at the master’s degree level. DIF: Knowledge REF: pp. 461-462 3. What characteristics are associated with the clinical nurse leader? (select all that apply) a. Is a generalist prepared at the master’s level. b. Provides care at the bedside for complex patients. c. Identifies the best resources at the lowest price to achieve best health outcomes for a client. d. Serves as care coordinator whose practice is limited to acute care settings. e. Is an advanced practice nurse with an earned doctorate in nursing. ANS: A, B Clinical nurse leaders are prepared as generalists at the master’s level. Clinical nurse leaders provide care to complex patients, change care plans as needed, and coordinate care to a group of patients. DIF: Comprehension REF: pp. 460-461 Chapter 27: Job Search: Finding Your Match Cherry & Jacob: Contemporary Nursing: Issues, Trends, and Management, 7th Edition MULTIPLE CHOICE 1. When should a follow-up letter be written to a recruiter? a. Before the interview to thank him or her for granting you the interview b. Approximately 2 weeks after you did not get the job to inquire c. A few days after the interview d. When you are sure you got the position ANS: C A follow-up letter thanking the recruiter is a courtesy and a reminder of the nurse’s interest in receiving a timely response. DIF: Knowledge REF: p. 472 2. Cover letters: a. should signify an interest in the employer and let the employer know that you took time to research the organization. b. must be handwritten so the employer can determine whether charting will be legible. c. should be limited to one paragraph because time is of the essence. d. let the employer know your desired salary, shift, and area of practice. ANS: A The purpose of the cover letter is to show the recruiter that the applicant has an interest in the facility and that he or she did homework to learn more about the facility. DIF: Knowledge REF: p. 471 3. A nurse interested in an entry level position as a registered nurse prepares the cover letter. A correctly written cover letter might include: a. “I am interested in a position where community outreach is valued as noted in your organization’s mission statement.” b. “I am highly educated and have several degrees including an Associate Degree in accounting, a certification in word processing, a Bachelor’s degree in journalism, and a Bachelor’s degree in nursing.” c. “I am looking for a position that will allow me to develop my skills and still have a balanced home and work life.” d. “I am married, a Christian, a single parent, and due to the downturn in the economy will be willing to work any shift or nursing unit.” ANS: A Letting the potential employer know you have researched the organization shows a sincere interest. DIF: Application REF: pp. 471-472 4. To determine basic competency, the interview for a nursing position may include a test in: a. pathophysiology. b. correctly writing nursing diagnoses. c. pharmacology. d. computer skills. ANS: C Preemployment pharmacology testing is common. Institutions often give a quiz to assess basic knowledge of routinely administered medications, their purposes, and associated adverse effects. DIF: Knowledge REF: p. 483 5. When the job applicant is asked to identify weaknesses, one appropriate reply for the novice nurse might be: a. “I have no weaknesses.” b. “Are you trying to set me up?” c. “Delegating to peers.” d. “Caring requires weakness.” ANS: C It is best to be honest about weaknesses the applicant may have. The employer does not expect the graduate nurse to have only strengths. All nurses can improve in their profession. DIF: Application REF: pp. 478-479 6. A valuable tool for determining whether a potential employer is suitable for your career goals is to: a. call The Joint Commission (TJC) to ask whether the organization meets required standards. b. assess the work climate by walking through the facility. c. review financial statements of the institution. d. ask what they can do for you rather than what you can do for them. ANS: B By observing the staff when taking a tour of the unit, the applicant can get an accurate feel for the culture and personality of the nursing unit. The applicant can assess the manager’s interactions with staff and can see how nurses respond to each challenge. DIF: Application REF: p. 479 7. Many nursing professionals have portfolios that include: a. family pictures. b. letters of commendation. c. articles that describe lack of quality in health care. d. high school achievements. ANS: B A portfolio includes letters from supervisors and patients, congratulatory messages from peers, and evaluations documented by supervisors. DIF: Knowledge REF: p. 474 8. A potential applicant reviews the application process and learns that preemployment physicals often include: a. cholesterol screening. b. pregnancy testing. c. urine drug screening. d. vision and hearing tests. ANS: C A preemployment physical examination is often required, and this includes routine blood tests, urine drug screening, and sometimes a chest radiograph. DIF: Knowledge REF: p. 483 9. A graduate nurse should begin the job search knowing that: a. few jobs are available for RNs because of a failing health care economy. b. it would be more promising to apply to a rural hospital than to an urban hospital. c. although efforts have been made to reduce expenditures for health care, the need for RNs remains stable. d. the demand for nurses to work in acute care hospitals has decreased because most services are being shifted to outpatient and ambulatory settings. ANS: C The nursing shortage has caused institutions to find unique ways to lure nursing graduates to their place of employment. These include sign-on bonuses, expense-paid weekend visits to the facility, tuition reimbursement, and low-interest loans. Although health care costs are a concern, attracting nurses is currently a priority, with little worry about associated costs. DIF: Comprehension REF: pp. 468-469 10. While performing a job search, a nurse reviews the list of Magnet hospitals of the American Nurses Credentialing Center because: a. these hospitals are able to hold on to revenue gained from patient care. b. nursing longevity, autonomy, and self-governance are important features of these institutions. c. all RNs have a minimum of a master’s degree and are credentialed in the area in which they provide care. d. these hospitals have the most up-to-date diagnostic equipment such as magnetic resonance imagery. ANS: B Magnet hospitals demonstrate excellence in areas such as low RN turnover rates, adherence to standards of nursing care as defined by the American Nurses Association, and mechanisms in place for staff participation in decision making. DIF: Comprehension REF: p. 469 11. According to the Hallmarks of the Professional Nursing Environment, a nurse should ask a potential employer: a. how salaries compare with those of larger facilities. b. what benefits are provided to RNs that are not available to other health care providers. c. what role nurses have in determining the quality of care. d. how the employer plans to provide long-term economic security to RNs. ANS: C The hallmark that suggests the role nurses have in determining the quality of care is noted in the philosophy of clinical care. DIF: Comprehension REF: p. 469 12. A nurse entering the job market wants to be certain that the first position is rewarding and allows for growth as a registered nurse. According to the Hallmarks of the Professional Nursing Environment, the nurse should ask: a. about the philosophy related to clinical care to determine the role of RNs in deciding outcomes related quality outcomes. b. if professional development opportunities are supported through paid leave and tuition reimbursement. c. how compensation packages compare to others in the same area and nationally. d. what the projected need for RNs is in the next 5 years and current turnover rate. ANS: A The hallmark that suggests the role nurses have in determining the quality of care is noted in the philosophy of clinical care. DIF: Application REF: p. 469 13. A nurse who is applying for a position contacts an expert on writing résumé and cover letters and is told: a. to use correction fluid sparingly. b. that when an error is made, a single line should be used to cross through the mistake, and initials should be inserted above the error. c. to limit margins to one half inch so that the page appears full of information. d. to be concise, limit the resume to a single page. ANS: D A resume is an effective, compressed one-page summary of the nurse’s education and employment history. DIF: Knowledge REF: pp. 472-473 14. The appropriate procedure for addressing a cover letter when the applicant is unsure of the name and title of the person to whom the letter should be addressed is to: a. address the letter to “To Whom It May Concern.” b. address the letter to “Nurse Recruiter.” c. call the facility to inquire about the name and title of the person. d. leave the salutation blank if the name and the title are unknown. ANS: C The letter should be addressed to a specific person. If the person’s name or title is unknown, refer to a marketing brochure, or call the recruitment office and ask for the correct title and spelling of the person’s name. DIF: Application REF: p. 471 15. When preparing the education section of a resume: a. list high school, followed by the first college attended. b. include all colleges attended, even if a degree was not awarded. c. omit the address of the university unless requested by the potential employer. d. list in reverse chronologic order the names, dates, and addresses of universities that awarded degrees. ANS: D Details about education should include degrees and diplomas awarded, names and locations of schools awarding them, and graduation dates, starting with the most recent graduation and degree. DIF: Knowledge REF: p. 473 16. A new graduate arrives for the first interview feeling prepared and excited about beginning a nursing career but quickly becomes concerned when the recruiter asks which illegal question? a. “Have you been convicted of a crime other than a minor traffic violation?” b. “We are very interested in hiring you but you do understand the position is contingent on the results of the preemployment physical and background check?” c. “What do you consider to be your major weaknesses for this particular position?” d. “Because you are applying for a position in the operating room, you must be able to be on call. How many children do you have responsibility for at home?” ANS: D It is illegal for the recruiter or interviewer to ask number of children or dependents prior to making a job offer. DIF: Application REF: p. 477, Box 27-6 17. A new graduate secures the first interview and wants to work where cultural diversity is appreciated. The applicant feels that accepting this interview was the right choice when the recruiter states, “We do not discriminate based on sex, race, or age and practice inclusiveness of diversity.” Which question would reflect this statement? a. “Nurses must work a master schedule where one month is Monday through Friday and the next month is the weekend shift. What is your religious belief about working on Sunday?” b. “We offer many benefits for employees such as educational benefits and health insurance.” c. “Our mission is to provide patient-centered care that reflects the belief of the patient and family with an aim to win them over to Western medicine and less superstition, don’t you agree?” d. “Do you have any issues with working with patients who are from a disadvantaged background because we have many people who refuse to work then want free care?” ANS: B Sharing information about employee benefits is not asking about discriminatory issues—it just informs the applicants of availability. DIF: Application REF: p. 477, Box 27-6 18. Which message would be appropriate to leave on answering systems? a. “You have reached 910-999-1212. Please leave your name and number and I look forward to returning your call as soon as possible.” b. Music that has messages that might be offensive to some callers downloaded as the response when applicant is unavailable. c. “Hi, I’m not here. Sorry you missed me. Call again.” d. “I am interviewing for positions, hope to be home soon. If you are calling about an interview, please leave your name, number, and possible positions available.” ANS: A A professional or appropriate message that lets the person know they have reached the correct number and that you will return the call as soon as available. DIF: Application REF: p. 477 MULTIPLE RESPONSE 1. When preparing a portfolio, the nurse should include: (select all that apply) a. copies of diplomas awarded by colleges/universities. b. copies of college transcripts. c. a personal photograph, if not included with the application. d. letters of recognition for scoring high on national achievement tests while in nursing school. e. a copy of the voter’s registration card. ANS: A, B, D Traditional documents, such as copies of diplomas, college transcripts, and recognition letters, are included in the portfolio. DIF: Knowledge REF: pp. 473-474 2. A new graduate nurse is preparing for an interview for her first position and knows that: (select all that apply) a. the primary goal for the first job is to complete orientation and should be considered as an extension of nursing school. b. appointments for interviews should be scheduled immediately after graduation to avoid rushing into a position. c. self-confidence can be improved with self-talk, which reminds the graduate that peers from her school are effective practitioners. d. work history of even nonmedical positions can demonstrate commitment and reliability so letters from these employers should be in portfolio. e. it is best to arrive 10 to 15 minutes before the scheduled interview dressed for success. ANS: C, D, E Internal dialogue establishes a means for the graduate to recall achievements that will lead to success in the first nursing position. Jobs such as cashier at a convenience store or dog-sitting can demonstrate responsibility such as handing cash or arriving on time. Arriving too early may cause the recruiter to feel rushed and arriving too late may demonstrate lack of time management and professionalism. DIF: Application REF: p. 476 3. A new graduate is excited about finding the first job but doesn’t want to make the costly mistake of taking the first job offered or taking a job that doesn’t meet her career goals. She learns the best way to know the culture of the potential employer is networking with: a. the executives at the competing agency. b. alumni that graduated from the same school and work in the area. c. nurses currently working in the agency. d. friends or people who have been patients at the agency. e. nurses who are former employees. ANS: B, C, D, E Networking is a valuable way to gain perspective on the work environment and culture of potential employers. Alumni that graduated from the same school and work in the area would most likely have your best interests in mind. Nurses working the agency have first-hand information. It might be best to ask more than one to get unbiased information. Friends or people who have been patients know the care received and often can determine the nurses’ attitude and pick up clues about the quality of patient contact. Nurses who are former employees have experienced the culture and can tell you from their perspective about the work culture and environment, but keep in mind that, if they left involuntarily, their perspective may be skewed. DIF: Comprehension REF: pp. 470-471 4. A new nurse decides to use the Internet to apply for possible positions. How does applying online compare to applying by more traditional methods? (select all that apply) a. Regardless of how one applies, it is important to check the resume for correctness. b. Follow-up with Internet applications is no sooner than 90 days compared to a 1-week follow-up for traditional applications. c. Researching the mission and philosophy of the agency is not necessary with Internet searches and applications. d. The resume and cover letter should be reviewed by someone with excellent skills in grammar and writing to ensure the first impression is one of professionalism and attention to detail. e. Internet applications should include specific questions to be answered prior to accepting an interview such as pay, shifts available, nurse-patient ratio, and autonomy of practice. ANS: A, D The resume and cover letter must be free of errors regardless if mailed, faxed, presented in person, or submitted online and both should be one page. The typical recruiter spends little time reviewing documents, so clarity and conciseness is important. DIF: Comprehension REF: pp. 470-471 5. A nurse preparing a resume for the first position is anxious to demonstrate all achievements, which included completing a rigorous accelerated program and maintaining a high GPA while also serving in many community projects and leadership positions. The applicant finds it impossible to include all the accolades on one page and researches tips for professional cover letters and resumes and learns: a. margins should be at least 0.5 inch on all four sides, making conciseness important. b. paper should be bright colored to catch the attention of the recruiter. c. legal size paper may be used when applicants have many accolades or extensive work history. d. standardized cover letters present a professional image as opposed to those that stress the applicant’s perspective too heavily. e. include in the cover letter that after researching other position, this is the applicant’s preference. ANS: A, E Ample margins—minimum of 0.5 inch (1 inch preferred)—should be used on all four sides to prevent appearing cluttered. A statement declaring this position is one’s first choice should be included. DIF: Comprehension REF: pp. 472-473 COMPLETION 1. The key to a successful interview is being . ANS: prepared Being prepared is the best way to feel confident about the interview and to be ready for questions that the interviewer may ask. Rehearsing potential questions will ease the applicant’s fears about how he or she should answer these questions. Dressing appropriately and arriving on time are behaviors that take preparation; these steps, if done well, will help the applicant do well during the interview. DIF: Knowledge REF: p. 476 MULTIPLE CHOICE 1. Graduates from approved schools of nursing cannot sign their charting as registered nurses (RNs) until they: pass the National Council Licensure Examination for Registered Nurses (NCLEX-RN®). b. provide evidence of mental competency. c. supply written proof of physical fitness. d. have signed an employment contract with a health care facility. ANS: A A compulsory license requirement must be met to legally practice or work as a registered nurse in any state or U.S. territory. Licenses are granted only after an applicant has successfully passed the NCLEX-RN examination. DIF: Comprehension REF: p. 486 2. Which statement concerning the National Council Licensure Examination for Registered Nurses (NCLEX-RN) examination is correct? a. Graduates from all three types of nursing programs (diploma, associate degree, and baccalaureate degree) take the same examination. b. The examination is scored on an interval scale rather than on a pass-fail basis. c. The examination is offered twice a year in major urban areas. d. The candidate has the option of choosing a pencil-and-paper format. ANS: A The purpose of the NCLEX-RN examination is to determine safe practice and the ability of candidates to perform at the entry level. Candidates from all three types of nursing programs must demonstrate the same competencies. DIF: Knowledge REF: p. 487 3. Computerized adaptive testing implies that: the candidate must be computer literate. competency is determined on the basis of difficulty of questions, knowledge of the nursing process, and the number of questions answered correctly. c. testing facilities have been adapted for the physically challenged candidate. d. questions cannot be adapted to the needs of the student. ANS: B Computerized adaptive testing is based on the measurement theory, by which the candidate must prove with a score of 95% that he or she is safe and knowledgeable at entry into the practice level. DIF: Comprehension REF: pp. 487-488 4. On the National Council Licensure Examination for Registered Nurses (NCLEX-RN) examination, when the candidate is asked to set goals in collaboration with other members of the health care team, the nurse is being tested in the area of: a. assessment. planning. c. analysis. d. implementation. ANS: B Setting goals is one of the first steps in the planning process. DIF: Application REF: p. 493 5. The length of the National Council Licensure Examination for Registered Nurses (NCLEX-RN) examination is based on the: a. last four numbers of the candidate’s social security number. b. location of testing. c. candidate’s educational preparation. d. performance of the candidate. DIF: Application REF: pp. 487-488 6. The primary purpose of the National Council Licensure Examination for Registered Nurses (NCLEX-RN) examination is to: ensure that practitioners have the minimum skills and knowledge needed to provide care that will produce the best patient care outcomes. b. regulate nursing education. c. determine the mandatory educational level required for nurses to practice. d. accredit schools of nursing. ANS: A The purpose of the NCLEX-RN examination is twofold: (1) to safeguard the public from unsafe practitioners and (2) to determine whether candidates can perform entry level skills. DIF: Comprehension REF: p. 487 7. A student nurse who is preparing to graduate and take the licensure examination asks, “What is compulsory licensure?” The appropriate response of the nursing advisor is which of the following? a. All candidates wishing to take the licensure examination must pass a drug screen. b. Candidates must not have a felony conviction. c. To practice as an RN, the nurse must be licensed as an RN. d. An impaired nurse must sign a legal document to acknowledge limitations on his or her practice. ANS: C Licensure is a prerequisite for practice to ensure public safety. DIF: Knowledge REF: p. 486 8. The National Council of State Boards of Nursing determines acceptable National Council Licensure Examination for Registered Nurses (NCLEX-RN) examination questions on the basis of: the geographic location of the candidate. research that indicates needed skills for positions in which most entry level nurses are employed. c. surveys conducted by employers to determine the weaknesses of entry level nurses. d. surveys of physicians performed to determine what nurses must know to provide safe care. ANS: B Periodically, the National Council of State Boards of Nursing surveys health care providers to identify nursing care activities of entry level nurses. DIF: Comprehension REF: p. 487 9. Although the NCLEX-RN examination has new formats for questions, the most common format is the: a. fill-in-the-blank item because candidates are not provided with clues from distracters. b. multiple-choice question item, which allows candidates to select the one correct answer. c. multiple-response item because these questions require a higher level of critical thinking. d. hot-spot item because these questions are written at the application level of Bloom’s analysis. ANS: B Most NCLEX-RN examination questions have three distracters and one correct answer, but the examination allows all four levels of Bloom’s taxonomy to be tested. DIF: Knowledge REF: pp. 488-489 10. When delegating care, the RN assigns one nurse to care for a patient with shingles and a different nurse to care for a patient with human immunodeficiency virus/acquired immunodeficiency disease syndrome (HIV/AIDS). This represents which category of nursing care? a. Safe and effective care environment b. Health promotion and maintenance c. Psychosocial integrity d. Teaching/learning ANS: A DIF: Analysis REF: p. 492 11. A patient is brought to the unit with mediastinal chest tubes with no fluctuation in the water seal chamber; arterial blood gas results reveal pH, 7.55; CO2, 55; HCO, 28 mEq/L, and O2, 98%. Carotid artery pulsation is visible with the head of the bed elevated and the use of tangential lighting. The first action of the nurse is to: The above question represents which level of Bloom’s taxonomy? a. Knowledge b. Comprehension c. Application d. Analysis ANS: D In analysis, the candidate must make a judgment call after determining relationships among data. DIF: Application REF: p. 491 12. A candidate who is taking the NCLEX-RN examination received only 75 questions before the test was stopped. She called her professor and stated, “I passed. I had to answer only 75 questions.” The professor correctly responds by saying: a. “You are now officially licensed; you answered the more difficult questions correctly.” “It is possible to receive only 75 questions and not be successful; however, we will keep a positive attitude.” c. “If you were given only 75 questions, you will have to retest because this is not enough to determine competency.” d. “You must have been extremely close to the passing standard because the computer shut off.” ANS: B Seventy-five questions is the minimum number of questions that can determine 95% competency of the candidate; however, receiving 75 questions can indicate that the candidate passed and was able to answer a broad range of questions covering the nursing process at higher cognitive levels, or it can mean that the candidate failed, answered even the lowest cognitive level questions incorrectly, and was unprepared in area/s of the nursing process. DIF: Application REF: pp. 487-488 13. On the basis of changes effective with the April 2010 test plan, candidates who take the NCLEX-RN examination: a. will have to answer more than 75 questions to be 95% certain that they are above the passing standard. b. will be required to submit a 100-word essay on an important nursing topic to evaluate safe nursing practice. should study and take practice examinations written at the application and analysis level to ensure that they can meet the higher standards of nursing care and health care delivery. d. will have additional time to complete the examination because most questions will be prepared in the alternate format, thereby requiring critical thinking. ANS: C In April 2007, the difficulty of the examination was increased, which would require higher cognitive level questions at the application and analysis level. DIF: Application REF: p. 490 14. In the National Council Licensure Examination for Registered Nurses (NCLEX-RN) examination test plan, client needs form the organizing framework of the examination with questions in four categories: safe and effective care environment, health promotion and maintenance, psychosocial integrity, and physiologic integrity. Certain processes are then integrated throughout the categories of client needs. Which process is integrated into all client need categories? a. Teaching/learning b. Health promotion c. Infection control d. Pharmacology ANS: A DIF: Comprehension REF: p. 492 15. Which action would help a student successfully prepare for the National Council Licensure Examination for Registered Nurses (NCLEX-RN) examination? a. Make note cards that can easily be retrieved to list only facts. b. Avoid timing oneself while studying and when in the actual testing mode to decrease anxiety. c. Ask peers for help because they had the same resources; especially seek their help for difficult concepts requiring critical thinking. Practice taking NCLEX-RN examination–type questions and reviewing rationales for correct and incorrect answers. ANS: D Practicing helps the student learn how questions are presented and what level of difficulty should be anticipated. Reviewing rationales for the answers is a teaching/learning opportunity that enables the student to judge whether the rationale used to answer the question is valid. DIF: Application REF: p. 495 16. The nurse prepares to apply sterile gloves needed for a procedure. After introducing self and verifying patient information, the nurse performs hand hygiene. The nurse should open the outer package and then perform the following steps in order: (items on left will be moved in correct order on the right) Open inner package, taking care not to touch inner surface. Put the glove on the nondominant hand using the sterile gloved hand. Put glove on dominant hand by grasping folded cuff edge, touching only inside of cuff. Adjust each glove carefully by sliding finders under the cuffs. The above question represents which type of alternate-format question written at which level of Bloom’s taxonomy? a. Drag-and-drop item, comprehension b. Chart/exhibit item, analysis c. Multiple-response, application d. Hot-spot item, knowledge ANS: A DIF: Analysis REF: pp. 489-490 17. A graduate is preparing for the National Council Licensure Examination for Registered Nurses (NCLEX-RN) examination. Based on the latest practice survey, the candidate would focus the most amount of time on which content? a. Stages of grief b. Pharmacologic pain management c. Practices to promote rest and sleep d. Prioritization of workload to manage time effectively ANS: B The current NLN test blueprint indicates that pharmacologic and parental therapies account for between 12% and 18% of the examination. DIF: Application REF: p. 492 18. Which strategy would promote a high rate of success on the National Council Licensure Examination for Registered Nurses (NCLEX-RN) examination? a. Since the examination is time limited, skip questions when unsure of the answer and return later if time permits. b. Work quickly through the examination to answer as many questions as possible. c. Mark questions that may need to have the answer reviewed to make it possible to quickly go back and make change after answering all questions. Read the entire question and all possible answer options before selecting an answer. ANS: D Candidates should carefully read the entire question and all answers before making a selection. DIF: Comprehension REF: p. 488 19. A candidate is taking the NCLEX-RN examination when the computer turns off. The candidate was aware they had reached the 6-hour time limit. A count of completed questions had been recorded on the note pad, and 100 questions were answered. How will the examination be scored? The computer will analyze the last 60 questions and if above the passing standard, the candidate passes. b. The candidate will be administered an additional 50 questions to determine ability to reach the 95% confidence interval. c. If the time runs out, the candidate automatically fails the examination because it is impossible to determine if candidate is safe. d. The candidate has the option to complete a simulated examination to show competence. ANS: A Run-out-of-time rule: if the candidate has answered the minimum number of questions, the computer analyzes the last 60 questions to determine if these questions were above or below the passing standard. DIF: Comprehension REF: p. 488 20. 20. The order reads: Administer 500 mg of Kefzol (cefazolin sodium) Intramuscular. Using information located on the medication vial above, the nurse will administer how many mL? The above question represents which type of alternate-format question written at which level of Bloom’s taxonomy? a. Multiple-response item, comprehension level b. Hot-spot item, application level c. Chart/exhibit item, analysis level d. Drag-and-drop item, knowledge level ANS: C The chart/exhibit presents the candidate with a chart or exhibit with information needed to answer the question. The question is at the analysis level since the candidate must calculate the correct amount to be given using a label requiring dilution. DIF: Application REF: pp. 490-491 21. “A patient with end-stage renal disease has a potassium level of 7.5 mEq/L. Based on this laboratory result, the nurse interprets which symptom as significant prompting which action? a. Drowsiness, stimulate the patient every 30 minutes b. Confusion, ask the patient to state their name and date of birth c. Irregular heartbeat, evaluate the patient’s capillary refill d. Muscle cramps, elevate the affected limb” ANS: A It is important to remember that candidates have specified laboratory values for which they must know the normal range and clinical manifestations when values are high, low, or critical. This question addresses an alteration in body system homeostasis. The student must know the normal lab value for potassium and common symptoms, as well as the correct nursing action. DIF: Analysis REF: p. 491 22. 22. A patient presents with chest pain that increases when lying flat and low-grade fever. Assessment reveals muffled heart sounds, tachycardia, and 3+ edema in the lower extremities. The nurse asks the patient to learn forward, which eases respiratory efforts. The nurse then places the stethoscope at which area on the chest to facilitate auscultation? The above question represents which type of alternate-format question written at which level of Bloom’s taxonomy? a. Multiple-response item, comprehension level b. Hot-spot item, analysis level c. Chart/exhibit item, application level d. Drag-and-drop item, knowledge level ANS: B This is a hot-spot item; students are presented with a figure on which they must indicate the answer by clicking the mouse over the area. The student would place the mouse over the left sternal border. It is analysis level because the student must take data from many sources to provide a complete assessment of the patient’s condition. DIF: Analysis REF: p. 489 OTHER 1. The nurse prepares to apply sterile gloves needed for a procedure. After introducing self and verifying patient information, the nurse performs hand hygiene. The nurse should open the outer package and then perform steps in which order. Put a comma and space between each answer choice (e.g., a, b, c, d). a. Open inner package, taking care not to touch inner surface. b. Put the glove on the nondominant hand using the sterile gloved hand. c. Put glove on dominant hand by grasping folded cuff edge, touching only inside of cuff. d. Adjust each glove carefully by sliding finders under the cuff. The nurse should open the inner package first, then put the glove on the dominant hand before the nondominant hand before adjusting the gloves. DIF: Application REF: pp. 489-490 [Show More]

Last updated: 1 year ago

Preview 1 out of 106 pages

Add to cart

Instant download

document-preview

Buy this document to get the full access instantly

Instant Download Access after purchase

Add to cart

Instant download

Reviews( 0 )

$15.00

Add to cart

Instant download

Can't find what you want? Try our AI powered Search

OR

REQUEST DOCUMENT
99
0

Document information


Connected school, study & course


About the document


Uploaded On

Mar 26, 2021

Number of pages

106

Written in

Seller


seller-icon
Good grade

Member since 4 years

397 Documents Sold


Additional information

This document has been written for:

Uploaded

Mar 26, 2021

Downloads

 0

Views

 99

Document Keyword Tags

Recommended For You

Get more on EXAM »

$15.00
What is Browsegrades

In Browsegrades, a student can earn by offering help to other student. Students can help other students with materials by upploading their notes and earn money.

We are here to help

We're available through e-mail, Twitter, Facebook, and live chat.
 FAQ
 Questions? Leave a message!

Follow us on
 Twitter

Copyright © Browsegrades · High quality services·